0% found this document useful (0 votes)
13 views56 pages

CLAT 2024 Mock Test with Solutions

The passage discusses Diogenes of Sinope's views on masturbation and autonomy. It mentions that Diogenes was fascinated by the causes of masturbation and believed that if everyone practiced it, the world would be a better place without wars. The passage also provides historical context about figures like Apollo, Hermes, and the nymph Echo in relation to masturbation.

Uploaded by

ks0090824
Copyright
© © All Rights Reserved
We take content rights seriously. If you suspect this is your content, claim it here.
Available Formats
Download as PDF, TXT or read online on Scribd
0% found this document useful (0 votes)
13 views56 pages

CLAT 2024 Mock Test with Solutions

The passage discusses Diogenes of Sinope's views on masturbation and autonomy. It mentions that Diogenes was fascinated by the causes of masturbation and believed that if everyone practiced it, the world would be a better place without wars. The passage also provides historical context about figures like Apollo, Hermes, and the nymph Echo in relation to masturbation.

Uploaded by

ks0090824
Copyright
© © All Rights Reserved
We take content rights seriously. If you suspect this is your content, claim it here.
Available Formats
Download as PDF, TXT or read online on Scribd

BATCH - CLAT ACHIEVERS 2024

Mock Test [CLAT 2024] #03


[with Hints and Solutions]
DATE:24/06/2023

Duration: 120 Minutes Candidate Name: _______________


Max. Marks: 120 Admit Card No. : _______________

INSTRUCTIONS TO CANDIDATES

1. The question booklet (QB) contains 120 multiple-choice questions.


2. Write your Admit Card No. on the first page of the QB before starting the test.
3. Use the separate carbonized Optical Mark Reader (OMR) Response Sheet to
answer all the questions. Make sure to read the detailed instructions provided
with the OMR Response Sheet before starting the test.
4. If you notice any discrepancy in the QB, request the invigilator to replace both
the QB and OMR Response Sheet. Do not use the previous OMR Response
Sheet with the new QB.
5. During the test, write the QB No., OMR Response Sheet No., and sign in the
designated space/column on the Attendance Sheet provided.
6. Keep your Admit Card signed by the Invigilator, as it will be required during
the admissions process.
7. The QB for the Undergraduate Five-Year Integrated Programme is worth 120
marks. Each correct answer earns 1 mark, and each wrong answer results in a
deduction of 0.25 marks. There are no deductions for unanswered questions.
8. You can keep the QB and the candidate's copy of the OMR Response Sheet
after the test.
The use of any unfair means, including the possession of electronic devices
such as mobile phones, headphones, and digital watches, is strictly prohibited
during the test. Impersonation or any other fraudulent practice may be a
criminal offense and will lead to disqualification and possible legal action.

[ 1]
CONTENTS OF QUESTION PAPER

Subject Q. Nos. Page No.


English Language 1 – 25 3–6
Logical Reasoning 1 - 25 7 – 12
Current Affairs Including General Knowledge 1 - 30 13 – 17
Legal Reasoning 1 - 30 18 – 26
Quantitative Techniques 1 - 10 27-28
Answer Key 1-120 29
Hints & Solution 1-120 30- 56

[ 2]
English Language
PASSAGE I
The orator Dio Chrysostom (c. 40–120 CE) mentions 3. Which of the following can be a suitable title for the
a fascinating fact concerning the Cynic philosopher passage?
(A) Diogenes' Fascination with Masturbation: A
Diogenes of Sinope (404–323 BCE) in his sixth
Crude Joke or Deeper Meaning?
Discourse. Dio informs us that Diogenes loved the (B) The Ecological Gesture of Diogenes:
fable about Pan discovering masturbation in Masturbation as Autonomy
particular. Due to his public demonstrations, Diogenes (C) Diogenes of Sinope: A Cynic Philosopher's
was already well-known for the practice by the time Unconventional Beliefs
Dio arrived. When asked about it, he is supposed to (D) The Trojan War and the Potential Impact of
have responded, "If only it were so simple to relieve Autonomy in Diogenes' Philosophy
hunger by rubbing one's stomach." Another time, in
4. Which of the following can be a suitable antonym
self-defense, he remarked, "Even fish are more from the passage to the word "nymph" except?
sensible than humans when they need to ejaculate." (A) Sprite (B) Human
They simply go outside and touch their skin against (C) Specter (D) Dryad
something abrasive. However, Diogenes' fascination
with the causes of masturbation offers us more than 5. Which one of the following is an incorrect statement
simply an excuse for a crude joke. I'd like to propose according to the passage?
(A) Diogenes of Sinope believed that practicing
that it's a significant ecological gesture.
masturbation could lead to a better world.
Apollo, according to legend, learnt the method from (B) Apollo learned the method of masturbation from
his father, Hermes, and later passed it on to shepherds his father, Zeus.
to assist them deal with having to spend so much time (C) Diogenes claimed that if everyone demonstrated
alone in the highlands. He was apparently irritated by autonomy through masturbation, the world would
his fruitless pursuit of the nymph Echo. Diogenes be a better place.
(D) According to Dio Chrysostom, Diogenes'
claimed that if everyone demonstrated this autonomy,
fascination with the causes of masturbation was
the world would be a better place. For starters, there solely for crude jokes.
wouldn't have been a Trojan War since philandering
Paris wouldn't have needed an excuse to flee with PASSAGE II
Helen. As Dio restates Diogenes' argument: India's nuclear capacity was one of the worst-kept
In reality, obtaining what people have put the greatest secrets in the international strategic community
effort and money into is the easiest and cheapest task between May 18, 1974, and May 11, 1998. The
establishment in New Delhi was constrained by the
of all. This is the item that has led to the destruction of
ethical conundrum of becoming a nuclear power
numerous cities and the pathetic collapse of numerous despite performing a nuclear bomb test. There was
nations. anxiety over India's announcement of nuclear weapon
capabilities because it had long been an advocate for
1. Which of the following can be inferred from the nonproliferation in the international world. Concerns
passage, except? also existed about how India would handle the
(A) Diogenes of Sinope believed that masturbation is international response, which included economic
sanctions, and that the ensuing arms race with
a form of autonomy and self-sufficiency.
Pakistan may enable the latter to level the playing
(B) Diogenes' fascination with the causes of field. That anxiety was allayed at Pokhran exactly 25
masturbation is merely a crude joke without any years ago. And it is now obvious that the calculated
deeper significance. risk taken by the Atal Bihari Vajpayee administration
(C) Diogenes argued that if everyone practiced paid off.
masturbation, it would lead to a better world. The US-led economic sanctions following the tests on
May 11 and May 13 did not have the terrible
(D) Diogenes believed that obtaining what people
repercussions they once may have since the Indian
have put great effort and money into is often the economy had expanded enough following
easiest and cheapest task. liberalization to be able to withstand them. On a
strategic level, the civil nuclear agreement, which was
2. Which of the following can be the tone of the promoted by George Bush Jr. and Manmohan Singh,
passage? made India a de facto nuclear power. The two nations
(A) Humorous (B) Ecological have gotten closer since then. On the other hand,
(C) Philosophical (D) Satirical Pakistan was viewed as an unstable nuclear power and

[ 3]
there was genuine danger that extremist organizations 10. Which of the following would best substitute the
may obtain access to its nuclear weapons.The fallout word, "nonproliferation" as used in the passage ?
from Pokhran II also caused India to reevaluate its (A) Nuclear testing
approach to the world because China blocked India's (B) Military expansion
membership in the Nuclear Suppliers Group, (C) Nuclear power development
undermining the idea that the US and the West were (D) Disarmament
the only ones to have double standards when it came
to non-proliferation. The fallout from Pokhran II also PASSAGE III
caused India to reevaluate its approach to the world The likening of cancer to a hostile enemy goes back
because China blocked India's membership in the more than a century. „To have a better understanding
Nuclear Suppliers Group, undermining the idea that of the dread thing we know as cancer just compare it
the US and the West were the only ones to have with the war,‟ wrote Cleveland‟s commissioner of
double standards when it came to non-proliferation. health, R H Bishop, Jr, during the First World War.
The development of India's nuclear arsenal also had He continued: „Prussianism might well be called the
an impact on its hostile neighbors. By maintaining a cancerous growth that is trying to kill the other
credible deterrent with regard to China, it has helped nations of Europe, for cancer, of itself, is a lawless
to balance the military disparity. growth of body cells which destroys life if allowed to
run its course.‟In the early 20th century – when
6. Which of the following conclusion from the passage patients were called „victims‟, long before remissions
can not be drawn ? became tenable for many – the idea of exercising a
(A) India's nuclear tests in 1974 and 1998 caused warlike campaign against cancer gained traction
economic sanctions that severely impacted its among philanthropists and expert physicians. Yet in
economy. their day-to-day lives, people rarely „battled‟ cancer.
(B) The civil nuclear agreement between the US and For thousands affected, fighting was not an option.
India made India a de facto nuclear power. Most died within a year or two of a malignant
(C) India's nuclear arsenal has contributed to diagnosis. Cancer patients and physicians had little
maintaining a military balance with China. information, scant diagnostic tests, and few
(D) The fallout from India's nuclear tests affected its medications to consider. For the most part, people
relationship with China and hindered its with cancer could not be helped except, for the lucky
membership in the Nuclear Suppliers Group. ones, by painkillers.In today‟s more promising
climate, a „fighting spirit‟ appeals to many people
7. Which event alleviated the anxiety surrounding India's with cancer. For some cancer patients I‟ve known, it‟s
nuclear weapon capabilities? been important to say – and for those around them to
(A) India's membership in the Nuclear Suppliers hear – that they‟re trying to summon their emotional
Group and physical powers, to do whatever they can to live
(B) The civil nuclear agreement between India and longer. Some deploy combative phrases to give
the US themselves pep talks („You can beat this‟) as
(C) The imposition of economic sanctions by the US encouragement before and while receiving cancer
(D) Pakistan's acquisition of nuclear weapons treatment, even though they know full well that the
outcome is beyond their control.
8. What impact did China's actions have on India's Yet hawkish words – talk of „battling‟, as opposed to,
approach to the world? say, „coping with‟ cancer – have fallen out of favour
(A) It led to India's economic expansion following among physicians, psychologists and patient
liberalization. advocates. As a practising oncologist I avoided that
(B) It enabled India to balance the military disparity sort of language. War metaphors seemed inapt for
with China. describing research or cancer care. And I recognised
(C) It reinforced the idea that the US and the West this risk: if a treatment doesn‟t work, if a tumour
had double standards in non-proliferation. progresses, patients who have been led to believe that
(D) It caused India to advocate for nonproliferation in they‟re supposed to put up a fight against cancer may
the international world. blame themselves, mistakenly thinking that they
lacked sufficient strength or will, when it‟s the
9. What concern existed regarding Pakistan's nuclear treatment that failed.
weapons?
(A) The fear of economic sanctions from the US 11. What is the meaning of the word "remissions" as used
(B) The potential arms race with India in the passage?
(C) The ethical conundrum faced by the government (A) Temporary improvement in cancer symptoms.
in New Delhi (B) Cancer treatment to remove tumors.
(D) The risk of extremist organizations gaining (C) Recovery from cancer.
access to the weapons (D) Painkillers prescribed for cancer patients.

[ 4]
12. Which of the following statements supports the main began to reconsider his government‟s position
idea of the passage? regarding the Russian Orthodox Church, and
(A) Cancer patients in the early 20th century had few eventually to the entire question of the role of religion
treatment options.
in an atheist empire. At this meeting, Stalin presented
(B) The likening of cancer to a hostile enemy is a
common metaphor. these men with a bold proposal: the same Soviet state
(C) Patients with cancer need to adopt a fighting that had destroyed their Church was now going to
spirit to beat the disease. devote its resources to bringing it back.
(D) War metaphors are inappropriate for describing The story of this meeting and the proposal to revive
cancer treatment. Orthodoxy is rarely told – but, when it is mentioned, it
is dismissed as a wartime measure, as temporary as
13. What is the tone of the passage? the friendship campaigns between the USSR and the
(A) Neutral (B) Optimistic
United States that also characterized the war years.
(C) Pessimistic (D) Hopeful
Yet to gloss over this meeting is to miss its
14. What figure of speech is used in the phrase "lawless significance as a shift in the Soviet approach to
growth"? religion, one that would leave a mark on religious life
(A) Metaphor (B) Smile for Soviet people and their descendants in the decades
(C) Personification (D) Hyperbole that followed.

15. Which of the following phrases is an example of a 16. What is the significance of the meeting that Stalin
"hawkish" word or phrase? called in September 1943?
(A) Coping with cancer. (A) It was the first time that Stalin met with
(B) Trying to summon emotional and physical Metropolitan Sergius, Metropolitan Aleksey, and
powers. Metropolitan Nikolay.
(C) Doing whatever they can to live longer. (B) It marked a shift in the Soviet approach to
(D) Putting up a fight against cancer religion.
(C) It was a temporary measure to establish
PASSAGE IV friendship campaigns between the USSR and the
In September 1943, as the tide of the Second World United States.
War was turning in the Soviet Union‟s favor, the (D) It was a meeting to discuss the Soviet Union's
Soviet leader Joseph Stalin called a meeting at the victory in the Second World War.
Kremlin. Alongside the foreign minister Vyacheslav
Molotov and the head of the secret police Vsevolod 17. What was the state of the Russian Orthodox Church
Merkulov were three men in Stalin‟s office for the before the meeting in September 1943?
first time: Metropolitan Sergius, Metropolitan (A) The Church was thriving and had a significant
Aleksey, and Metropolitan Nikolay, three of the few role in society.
Orthodox Church hierarchs left in the Soviet Union. (B) The Church was heavily persecuted by the Soviet
The fact of such a meeting taking place is naturally state.
surprising. Even those who know little about the (C) The Church had already been revived by the
Soviet Union are familiar with its anti-religious Soviet government.
policies, especially thanks to Cold War rhetoric about (D) The Church was a minor religious institution with
„godless communists‟. Indeed, this meeting was being little influence.
held after decades of persecution of the Russian
Orthodox Church by the officially atheist Soviet state. 18. How did Stalin's proposal to revive Orthodoxy affect
The three Metropolitans came to this meeting after the Soviet approach to religion?
decades of watching their Church decimated all (A) It led to a permanent restoration of religious
around them. As they greeted Stalin at the Kremlin, freedom in the Soviet Union.
many of their fellow clergy were imprisoned in labour (B) It had no impact on the Soviet approach to
camps – and others were dead. By the end of the religion.
1930s, the Soviet state had effectively destroyed much (C) It led to a temporary reconciliation between the
of the official existence of what had been for centuries Soviet state and the Church.
imperial Russia‟s most powerful and wealthy religious (D) It intensified the Soviet state's anti-religious
institution. policies.
But in September 1943, as Stalin imagined a role for
the victorious Soviet Union in a postwar world, he
[ 5]
19. What was the purpose of the meeting in September Revolution (1905-11) or the authoritarian shahs of the
1943? Pahlavi dynasty (1925-79). Drawing on modern
(A) To discuss Soviet victory in the Second World literary histories of Persian written by both Iranians
War. and Indians, they refashioned Persian into a national
(B) To establish friendship campaigns between the language, and Persian literature into a national
USSR and the United States. heritage, positioning Iran as the proprietor of the
(C) To revive the Russian Orthodox Church. Persianate tradition
(D) To persecute the remaining Orthodox Church
hierarchies. 21. Which of the following is the best synonym for the
word "sclerotic" as used in the passage?
20. What is the tone of the passage? (A) Hardened (B) Flexible
(A) Optimistic (B) Cynical (C) Elastic (D) Soft
(C) Neutral (D) Dramatic
22. What is the main idea of the passage?
PASSAGE V (A) Cultural exchange is always progressive
Cultural exchange is often assumed to be progressive, (B) Indo-Iranian exchange fostered modern
nationalism
but it has neither an inherent politics nor an inevitable
(C) Persian was transformed into the national
outcome. As nationalism rises across the globe, many language of Iran in the 19th-century
see „cultural exchange‟ as the antidote to nationalist (D) Nationalism is rising across the globe.
xenophobia. Such exchange was, in fact, an integral
part of the emergence of national literatures, cultures 23. Which of the following statements is supported by the
and identities in Iran and South Asia. Rather than passage?
resulting in greater cosmopolitanism, the Indo-Iranian (A) Persian has always been the national language of
Iran
exchange fostered modern nationalism.
(B) Iranian intellectuals in the 19th century
Today, Persian is the national language of Iran. transformed Persian into a national language
Although it has a long history there, its national status (C) Cultural exchange leads to greater
and close association with the country depend on cosmopolitanism
events in the 19th century. As the Russian Empire (D) Nationalism is declining across the globe
swallowed up swathes of Iranian territory in the 1810s
and ‟20s, and the sclerotic Qajar dynasty ruling Iran 24. What is the tone of the passage?
(A) Objective (B) Critical
proved incapable of resistance, Iranian nationalism
(C) Sarcastic (D) Ironic
emerged to challenge both foreign imperialism and
local despotism.Photograph of the Shah Mosque, 25. Which figure of speech is used in the following
Persia, in the late 19th century. Courtesy of the V&A sentence: "As the Russian Empire swallowed up
Museum, London swathes of Iranian territory in the 1810s and ‟20s, and
By the end of the 19th century, Iranian intellectuals the sclerotic Qajar dynasty ruling Iran proved
incapable of resistance, Iranian nationalism emerged
were transforming Persian from the lingua franca of a
to challenge both foreign imperialism and local
centreless Persianate world spanning much of Asia despotism"?
into the language of national identity in the nation- (A) Metaphor (B) Simile
state of Iran. Nationalist modernizers made reforming (C) Personification (D) Allusion
the Persian language a central project, whether they
were democratic revolutionaries of the Constitutional

[ 6]
Logical Reasoning
PASSAGE I (B) The therapeutic effects of swinging have been
Over time, the swing‟s uses slightly changed as it well-established and universally accepted by
appeared in different cultures. It became an medical professionals throughout history.
instrument of play and discipline. Swinging can (C) Swinging has undergone changes in its cultural
produce different, even contradictory, experiences in significance over time and has been associated
similar ways to the practice of „blanket tossing‟, in with both positive and negative experiences.
which a person is punished or celebrated by being (D) The fear of falling associated with swinging has
tossed into the air and caught on an open blanket prevented it from being widely enjoyed in
held taut by a group of people. many cultures throughout history.
Even when it wasn‟t used punitively, the swing still
produced undesirable effects. On many occasions, 2. Based on the given passage, which of the following
and in essentially pedestrian societies, those who can be assumed about swinging machines used for
swung often experienced vertigo and dizziness. the treatment of various forms of insanity?
Others experienced a fear of falling as they swung, (A) The patients who were subjected to swinging
either due to an overly long rope or a saddle that machines never experienced physical
threatened to break. Swinging was not always a discomfort.
positive experience. Well until the end of the 18th (B) Swinging machines were popular only in the
century, European and American doctors worked parks and fairs of Bohemia.
with this discomfort, believing that the sweating, (C) Swinging machines were invented in the 1820s
retching or vomiting that accompanied swinging by the Czech anatomist Jan Evangelista
could be therapeutic. In the 1820s, the Czech Purkyně
anatomist Jan Evangelista Purkyně, known for (D) Swinging machines were believed to have
delving into the labyrinth of the inner ear, confessed therapeutic benefits by European and American
that he had suffered unspeakably while subjected to doctors until the end of the 18th century.
the rigours of swinging. To explore the ear‟s
labyrinth, the Czech sage had set up a rotating chair 3. What can be inferred about the historical belief
suspended by a rope, a device not unlike those used regarding the therapeutic effects of swinging?
at the time for the treatment of various forms of (A) European and American doctors believed that
insanity, and very similar to the swinging machines swinging could cure various forms of insanity.
that were becoming popular in the parks and fairs of (B) Swinging was never considered as a
Bohemia. After swinging in his device for an hour therapeutic activity in the past.
and a half, he described his suffering as unbearable. (C) Jan Evangelista Purkyně was the only doctor to
But swinging has not only been a source of physical experience the negative effects of swinging.
discomfort. It‟s also a source of horror. The history (D) The discomfort caused by swinging was always
of spiritualism contains numerous references to considered a negative outcome.
swings (or pendulums). Sir Arthur Conan Doyle, the
creator of the Sherlock Holmes character, included 4. Which of the following statements strengthens the
one in his History of Spiritualism (1925), and argument that swinging was initially considered a
Victorian photographers, so prone to portraits of the device of death that became apotropaic?
dead, depicted apparently dead girls on swings. The (A) In some cultures, swinging was used as a form
trope of a „haunted swing‟ that moves on its own of punishment for criminals.
would reappear in the horror films and amusement (B) The swing‟s uses changed over time, and it
parks of the 20th century. became an instrument of play and discipline.
(C) Athenians began swinging themselves on
1. What can be concluded about the historical and wooden planks hung from ropes to ward off an
cultural significance of swinging as described in the evil spell, which prevented young Athenian
passage? girls from hanging themselves.
(A) Swinging has always been a purely positive (D) Swinging machines were becoming popular in
experience for those who have engaged in it. the parks and fairs of Bohemia in the 1820s.

[ 7]
5. What is the main argument presented in the cause of the crime. Alternatively, he might explain
passage? that he will have a completely different set of
(A) Swinging was primarily used as a punishment circumstances than he previously did, emphasising
in ancient cultures and was not seen as a the contribution of the situation in bringing about a
positive experience until much later. crime. Which will give the parole board more
(B) The discomfort and negative effects associated confidence that the individual will not reoffend?
with swinging were believed to have When people tend to attribute wrongdoing more to
therapeutic benefits by European and American the person, arguments about complete personal
doctors until the end of the 18th century. change may be more persuasive.
(C) Swinging has evolved over time to become an
instrument of play and discipline, but it has also 6. Based on the passage, which of the following
been a source of physical discomfort and assumptions can be made about the fundamental
horror. attribution error?
(D) The swinging machine invented by Jan (A) People tend to attribute behavior to a person's
Evangelista Purkyně was used to treat various character rather than the situation because they
forms of insanity in the 19th century. lack information about the situation.
(B) The fundamental attribution error can lead to
PASSAGE II moral condemnation of individuals.
A classic idea in social psychology is that, in (C) The fundamental attribution error is more likely
seeking to explain someone‟s behaviour, people to occur in situations where people have a lot of
tend to inflate the importance of dispositions and information about the situational constraints.
neglect the importance of situations. The idea dates (D) The fundamental attribution error is only
to the beginning of the field, appearing in the work relevant in legal contexts.
of Kurt Lewin (1930), Fritz Heider (1944) and
Gustav Ichheiser (1949). In his 1977 paper, Lee 7. In a legal setting, which of the following scenarios is
Ross coined a term for it: the „fundamental most likely to lead to a reduced sentence for a
attribution error‟. Today, it is surely one of the best- convicted criminal?
known phenomenon names in the social sciences. In (A) The defense argues that the defendant had no
many experiments, participants have been found to choice but to commit the crime due to extreme
attribute behaviour to the personal dispositions of situational constraints.
individuals operating under sometimes ludicrously (B) The prosecution argues that the defendant's
prominent situational constraints. Some of the most character is fundamentally flawed and that they
striking evidence comes from the various „no are likely to reoffend.
choice‟ paradigms, where participants attribute an (C) The judge and jury attribute the crime to the
opinion to someone whom they know to have been defendant's disposition rather than the
ordered to express that opinion. situational factors.
(D) The defendant emphasizes that they have
What does it matter if we explain behaviour as
undergone significant personal growth since
sourced in the person versus the situation? Well, the committing the crime.
difference can be both morally and legally
consequential. Because character attributions are 8. Based on the passage, which of the following is a
judgments of a person‟s internal tendencies, they likely implication of the fundamental attribution
can be a powerful means of moral condemnation error in the legal system?
(eg, „That driver who cut me off is a complete (A) Criminals are more likely to receive harsher
sentences when situational factors are taken
asshole‟). By contrast, situational attributions can
into account.
help exculpate or excuse a transgressor (eg, „That (B) Judges and juries are more likely to consider
driver was probably under a lot of pressure‟). Since situational factors when making sentencing
our attributions reflect our understanding of what decisions.
caused a behaviour, they inform how it should be (C) The use of situational attributions in legal
dealt with, now and in the future. For example, a settings can lead to moral condemnation of the
convicted criminal might explain to a parole board defendant.
(D) Defendants who emphasize personal growth are
that he has completely changed since the time of his
more likely to receive reduced sentences.
offence; this emphasises the role of the person as the

[8]
9. What is the main argument presented in the of Life and Aurelius‟ Meditations – proves that
passage? these philosophers did not constantly live in the
(A) The fundamental attribution error is a well- moment. Just as speaking distracts yamabushi from
known phenomenon in social psychology that
their training and their ability to live in the moment,
affects how people explain behavior and has
significant moral and legal consequences. so does writing. In Shugendō, there is no place for
(B) Situational factors are the primary cause of self-reflection on daily problems through journaling.
behavior, and attributing behavior to personal We write not for the present, but for the future. We
dispositions can lead to moral condemnation write to reflect for a later version of ourselves or
and harsh legal consequences. others. We don‟t write to be in the moment.
(C) Personal dispositions are the primary cause of Likewise, reading can also take our minds away
behavior, and attributing behavior to situational from the world, away from concentrating on our
factors can lead to moral exoneration and training and, worst of all, away from the now. What
lenient legal consequences. would it mean to truly accept and deeply embrace
(D) The use of situational attributions in legal the possibilities of the present?
settings is the best way to exculpate
transgressors and promote moral rehabilitation.
11. What is the main difference between the Stoic
10. Based on the information in the passage, which of philosophy of living in the present and the Shugendō
the following is the best course of action for philosophy of living in the moment?
individuals to avoid the fundamental attribution (A) The Stoic philosophy emphasizes the
error in their moral and legal judgments? importance of accepting the past and future,
(A) Always prioritize situational factors over while the Shugendō philosophy focuses on the
personal disposition attributions. present moment only.
(B) Avoid making any moral or legal judgments (B) The Shugendō philosophy believes that writing
about individuals' behavior. and reading can distract one from living in the
moment, while the Stoic philosophy sees these
(C) Consider both situational factors and personal
activities as helpful for reflection and
disposition attributions in moral and legal acceptance.
judgments. (C) The Stoic philosophy emphasizes mindfulness
(D) Consciously overlook situational factors in and meditation, while the Shugendō philosophy
moral and legal judgments. focuses on physical training and walking
meditation.
(D) The Shugendō philosophy emphasizes living in
PASSAGE III
the moment through training and meditation,
The Stoic model of acceptance is built around living while the Stoic philosophy is incomplete in its
in the present and not worrying about a future or approach to living in the present.
past you cannot control. As Seneca wrote: „The
whole future lies in uncertainty: live immediately.‟ 12. What can be inferred about Shugendō philosophy
Shugendō training is also a practice of living in the and mindfulness practices?
moment. Walking meditation, what we call tosogyo, (A) Shugendō philosophy and mindfulness
practices are the same in terms of their
is one way we do this. It‟s impossible not to be in
approach to living in the moment.
the moment when you‟re walking on a precarious (B) Shugendō philosophy places more emphasis on
snowbank, where one wrong step could prove fatal. living in the moment than mindfulness
And through tokogatame meditation – toko means practices.
„ground‟, and gatame literally means „to harden or (C) Shugendō philosophy and mindfulness
solidify‟ – we learn to be in the moment, to solidify practices have some similarities in terms of
our place in the world. These practices might seem living in the present, but Shugendō places more
similar to current mindfulness practices that are value on activities that directly involve the
inspired by Stoicism. But the Stoic philosophy of present moment.
living in the present is incomplete. (D) Shugendō philosophy and mindfulness
According to Shugendō philosophy, Seneca‟s practices are fundamentally different in their
invitation to „live immediately‟ and Marcus approach to living in the moment.
Aurelius‟ phrase „confine yourself to the present‟
are both contradictory. That those phrases were
written down in books – Seneca‟s On the Shortness
[9]
13. Based on the passage, which assumption is Ideas of reason are concepts that gave up their
necessary for the Shugendō philosophy to reject the grounding in concrete, worldly experience to reach
Stoic model of living in the present? for the transcendent. Aesthetic ideas, by analogy, are
concepts that reach for the transcendent while
(A) Writing is a distraction from living in the
remaining grounded in experience, but give up on
present moment. being concepts. As the Romantics quickly noticed,
(B) Mindfulness practices are not effective for and Kant artfully avoids observing, this makes
achieving acceptance poetry a little too good at doing God‟s job. For Kant,
(C) The Stoic philosophy is incomplete. great poems take a concept – it could be a
(D) Living in the present moment is the only way to transcendent concept like „eternity‟ or „God‟ or an
achieve acceptance. interesting, normal concept like „envy‟ or „death‟ –
and drag it to a place halfway between pure reason
and experience, where concepts fall apart but the
14. Which of the following, if true, would strengthen the imagination itself turns into a form of reason.
argument that the Shugendō philosophy of living in Poetry, Kant says, is an ever-expanding panorama of
the present is more complete than the Stoic a concept‟s „implications and affinities with other
philosophy? concepts‟. It „opens the mind‟ to an „immeasurable
(A) Shugendō has a longer history than Stoicism. field of interrelated thoughts‟, each one concrete and
(B) Shugendō does not allow for any form of self- worldly, held together by the grace of an
unnameable pattern. Poetry reaches into the
reflection or journaling.
ineffable that binds the „mere words‟ of a concept,
(C) Shugendō training includes meditation be it „God‟ or „envy‟ or „dog‟, to its life in thought
practices that are more intense than those of and feeling.
Stoicism.
(D) Shugendō does not involve any form of reading
or writing. 16. Based on the passage, which of the following is true
about Kant's view on poetry?
15. Which of the following, if true, would weaken the (A) Kant believed that poetry is incapable of
argument that the Stoic philosophy of living in the conveying transcendent concepts like God or
present is incomplete according to Shugendō eternity.
philosophy? (B) Kant saw the poetic faculty as a substitute for
(A) The Stoic and Shugendō philosophies share the reason in its capacity to create aesthetic ideas.
belief that living in the present is important for (C) For Kant, the imagination's role in poetry is to
personal growth. provide a concrete grounding for transcendent
(B) Some Shugendō practitioners engage in self- concepts.
reflection and journaling to reflect on their (D) Kant believed that poetry is a tool for
training and personal growth. unravelling complex concepts into more
(C) Both Stoic and Shugendō practices involve a tangible and easily understood forms.
level of discipline and training to fully embrace
the present moment. 17. Based on the passage, what can be inferred about
(D) Shugendō philosophy does not have a complete Kant's view of the role of poetry in understanding
understanding of the principles of Stoicism. transcendent concepts?
(A) Poetry is a futile attempt to reach the
PASSAGE IV transcendent, as concepts are limited to
Baumgarten charmingly called the poetic faculty concrete, worldly experience.
„the analogue of reason‟ or „the reason below‟. Kant, (B) Poetry is the only way to fully comprehend
not to be outdone, argued that the poetic faculty is transcendent concepts, as reason alone cannot
the imagination‟s analogue to reason‟s special grasp their implications and affinities.
world-making, unimaginable thoughts. A work of (C) Poetry is a means of bridging the gap between
art, and most of all a poem, creates an aesthetic idea:
reason and experience in order to gain insight
„a representation of the imagination which prompts
a wealth of thought to which no concept can be into transcendent concepts.
adequate, and which no language can name.‟ As (D) Poetry is only capable of grasping concrete and
Kant observes: „An aesthetic idea is the counterpart worldly concepts, and cannot contribute to
of an idea of reason [God, self, logos], which is, understanding transcendent concepts.
conversely, a concept to which no intuition
(representation of the imagination) can be adequate.‟
[10]
18. How does the passage provide support for the And there are indeed some similarities between
argument that poetry plays a significant role in ancient magnates and their modern counterparts
understanding transcendent concepts according to when it comes to such public displays of generosity.
Kant? Through a closer look at the expenditures of the
(A) By explaining the limitations of reason in Roman elite in particular, though, we can discern the
comprehending transcendent concepts outlines of a historically distinctive modality of
(B) By discussing the relationship between public giving. It was Veyne who provided that
aesthetic ideas and concepts of reason closer look in the 800 pages of his masterwork, Le
(C) By providing examples of transcendent pain et le cirque: Sociologie historique d‟un
concepts explored in poetry pluralisme politique (1976), translated as Bread and
(D) By comparing the effectiveness of poetry and Circuses: Historical Sociology and Political
reason in understanding transcendent concepts Pluralism (1990).
The whole practice of public expenditure and civic
19. Which of the following, if true, would weaken the benefaction in the ancient Mediterranean world has
argument that poetry plays a significant role in come to be known as „euergetism‟, a neologism
understanding transcendent concepts according to derived from ancient Greek and meaning, literally,
Kant as presented in the passage? „the doing of good deeds‟. In the Roman Empire,
(A) Many great works of literature do not explore what euergetism meant in practice was the transfer
transcendent concepts. of wealth from the private to the public sphere. The
(B) Reason is the only reliable way to understand arena in which this transfer played out was the local
transcendent concepts. municipality. At the lower end of the scale of civic
(C) Transcendent concepts are beyond human benefaction was the organisation and funding of
understanding and cannot be explored through public banquets and public entertainments, such as
poetry or reason. gladiatorial combats. Higher-order benefactions
(D) Aesthetic ideas do not provide any new insight included the beautification of dilapidated public
into transcendent concepts beyond what can be buildings, the restoration of infrastructural works
achieved through reason alone. that had fallen into disrepair, and the establishment
of perpetual endowments. The truly big-ticket items
20. What is the author's view on the role of poetry in were the major public buildings financed by local
understanding transcendent concepts according to elites, the grand monuments that travellers to any
Kant? part of the Roman Empire still encounter today:
(A) The author believes that poetry is the only temples, theatres and amphitheatres, libraries, public
reliable way to understand transcendent baths, colonnades and basilicas. These structures
concepts. could be found, in different but recognisable shapes
(B) The author argues that poetry and reason are and sizes, from Rome and the dazzling metropolises
equally important in understanding of the East to the smaller cities and towns of North
transcendent concepts. Africa, Spain and Gaul.
(C) The author contends that poetry plays a
significant role in understanding transcendent 21. Based on the information provided in the passage,
concepts according to Kant. which of the following statements is true about the
(D) The author maintains that poetry has no role in practice of euergetism in the Greco-Roman world?
understanding transcendent concepts. (A) Euergetism was a practice exclusively limited
to the Roman Empire.
PASSAGE V
(B) The primary goal of euergetism was to transfer
The dangers of getting things wrong by assuming a
wealth from the public to the private sphere.
casual familiarity with antiquity are revealed when
(C) Public banquets and entertainments were the
we consider a topic of enduring fascination to
only forms of civic benefaction in the Greco-
ancient historians: the widespread phenomenon of
Roman world.
public giving and civic benefaction in the Greco-
(D) Major public buildings, such as temples and
Roman world. At a glance, this public giving looks a
amphitheatres, were financed by local elites as
lot like the contemporary practice of high-profile
part of euergetism.
charitable donations, the establishment of major
foundations, the patronage of the arts, and so on.
[11]
24. What can be inferred from the passage about the
22. Based on the information provided in the passage, motivations of the wealthy elite who engaged in
which of the following assumptions can be made euergetism in the Greco-Roman world?
about the relationship between euergetism in the (A) The wealthy elite engaged in euergetism
Greco-Roman world and modern-day philanthropy? primarily to gain personal fame and
(A) The primary goal of euergetism was to achieve recognition.
personal recognition and gain political power, (B) The wealthy elite engaged in euergetism
whereas modern-day philanthropy is primarily primarily to promote their own political and
driven by altruistic motives. social interests.
(B) The forms of public giving in the Greco-Roman (C) The wealthy elite engaged in euergetism
world, such as financing public buildings and primarily out of a sense of civic duty and public
banquets, were similar in nature and purpose to spirit
modern-day charitable donations and (D) The motivations of the wealthy elite who
establishment of foundations. engaged in euergetism cannot be determined
(C) Euergetism in the Greco-Roman world was a from the information provided in the passage.
spontaneous and voluntary practice, whereas
modern-day philanthropy is largely motivated 25. Which of the following, if true, would most weaken
by tax incentives and public relations benefits. the argument presented in the passage about the
(D) The practice of euergetism in the Greco-Roman unique nature of euergetism in the Greco-Roman
world was mainly driven by religious beliefs world?
and a desire to please the gods, whereas (A) Similar practices of public giving can be found
modern-day philanthropy is driven by secular in other ancient civilizations outside of the
concerns such as social justice and Greco-Roman world.
environmental protection. (B) The wealthy elite who engaged in euergetism
were primarily motivated by a desire for
23. Which of the following strengthens the argument personal fame and recognition.
that euergetism in the Greco-Roman world was a (C) Euergetism in the Greco-Roman world was not
historically distinctive modality of public giving? limited to the transfer of wealth from the
(A) The fact that similar forms of public giving, private to the public sphere.
such as financing public buildings and (D) The types of public buildings financed by the
banquets, were also prevalent in other ancient wealthy elite in the Greco-Roman world were
civilizations such as ancient Egypt and not significantly different from those financed
Mesopotamia. by wealthy elites in other ancient civilizations.
(B) The fact that the practice of euergetism was
largely confined to the wealthy elite, who used
it as a means of gaining political power and
influence.
(C) The fact that the major public buildings
financed by local elites in the Roman Empire,
such as temples and amphitheatres, were built
to serve specific civic and religious functions.
(D) The fact that euergetism was a major factor in
promoting social cohesion and civic identity in
the Roman Empire.

[12]
GK and Current affairs
PASSAGE I
1990s brought with them increased pressure to 5. Who was the Chief Scientific Adviser to the Indian
quickly develop nuclear weapons. With the fall of Prime Minister during the Pokhran-II tests in 1998?
the USSR in 1991, India lost one of its biggest (A) APJ Abdul Kalam
military allies, since the time Indira Gandhi had (B) R Chidambaram
signed a 20-year security pact with it in 1971. (C) Homi Bhabha
Furthermore, the US continued to provide military (D) Raj Verma
aid to Pakistan despite its own misgivings with its
nuclear weapons programme. For India, it felt like PASSAGE II
its window of opportunity was fast closing. Thus, in The National Youth Day is being celebrated on
1995, then Prime Minister PV Narasimha Rao January 12 to commemorate the birth anniversary
permitted the preparations for carrying out a nuclear of Swami Vivekananda. In 1984, the Indian
test in December 1995. However, logistical and government declared Swamiji's birth anniversary as
political reasons pushed back the tests further. National Youth Day and since 1985, it has been
After a few years of domestic turmoil when the celebrated to honor and recognize Vivekananda's
political will to conduct nuclear testing was teachings.
wanting, in 1998, the National Democratic Alliance In India, National Youth Day is celebrated to
(NDA) led by the BJP came to power under the promote awareness of people's rights and to provide
leadership of Atal Bihari Vajpayee. One of the key information about them. Keeping the ideals of
promises in its manifesto was to “induct nuclear Swami Vivekananda alive and motivating young
weapons” into India‟s arsenal, While the tests in people is the main objective of the celebration. A
1998 also invited sanctions from some countries number of programs are conducted across the
(like the US), the condemnation was far from nation on National Youth Day (Rashtriya Yuva
universal like in 1974. In context of India‟s fast- Diwas), including speeches, music, youth
growing economy and market potential, India was conventions, seminars, yoga asanas, presentations,
able to stand its ground and thus cement its status as essay writing, recitation competitions, and sports
a dominant nation state. competitions.
Vivekananda propounded the significance of
1. Which year did India conduct its first nuclear bomb education in nation building. He believed that
test in Pokhran? education is the primary means of empowering
(A) 1974 (B) 1973 people. He particularly emphasized education being
(C) 1994 (D) 1998 relevant to the common masses. One of the famous
proverbs of Swami Vivekananda among the youth
2. What was the code name of the first nuclear bomb is, “Arise! Awake! and stop not until the goal is
test conducted in Pokhran in 1974? reached.”
(A) Smiling Buddha
(B) Operation Shakti 6. What is the pre-monastic name of Swami
(C) Pokhran-I Vivekananda?
(D) Operation Parakram (A) Narendra Nath dutta
(B) Naga tilak dutta
3. How many nuclear bomb tests did India conduct in (C) Vivek Nanda
Pokhran in 1998 ? (D) None of the above
(A) Two (B) Three
(C) Four (D) One 7. What is the theme of Youth day 2023?
(A) Viksit Bharat-Viksit Yuva
4. What was the code name of the thermonuclear (B) The Youth, the strength
(C) Viksit Yuva-Viksit Bharat.
device tested by India in Pokhran in 1998?
(D) It's all in the mind
(A) Shakti-I (B) Shakti-II
(C) Shakti-III (D) Shakti-IV 8. When is International Youth day is celebrated?
(A) January 12 (B) August 12
(C) January 10 (D) August 14
[13]
14. In 2021, United Nations General Assembly
9. Vivekananda is best known for his speech at Which designated ____________(a) as the 'International
Parliament? day of Women Judges'. The theme for 2023
(A) World's Religions in Canada in 1893 (A) 23 September
(B) World's Religions in Mexico in 1893 (B) 9th August
(C) World's Religions in UK in 1893 (C) 10th March
(D) World's Religions in Chicago in 1893 (D) 8th March

10. Swami vivekananda is a disciple of? 15.


(A) Abhayadatta Sri (A) Conditions of (1) Article 129
(B) Chandrakirti appointment of HC
(C) Ramakrishna Paramhamsa Judge
(D) None of the above (B) Salaries, allowances (2) Article 127
etc. of the judges
PASSAGE III (C) Appointment of Ad- (3) Article 125
The government elevated two high court chief hoc Judges
justices to the Supreme court, taking it to its full (D) Supreme court to be (4) Article 217
strength of 34 judges after a gap of nine months. the court of record
Law Minister Kiren Rijiju took to Twitter to Option
announce the fresh appointments to the top court (A) A - 4 ; B - 2 ; C - 1 ; D - 3
and extended his best wishes to them. (B) A - 3 ; B - 1 ; C - 2 ; D - 4
Justice Rajesh Bindal, Chief Justice, Allahabad HC (C) A - 4 ; B - 2 ; C - 3 ; D - 1
and Justice Aravind Kumar, Chief Justice, Gujarat (D) A - 4 ; B - 3 ; C - 2 ; D -1
HC took oath as judges of the Supreme Court,
sources familiar with the process of appointing PASSAGE IV
judges to the higher judiciary said. The Department In a series of tweets, PM Modi shared the tweet by
of Justice in the Union law ministry issued separate Commerce and Industry minister Piyush Goyal
notifications announcing their appointments. about the launch of the PM MITRA Textile park in
UP today. He said, "Uttar Pradesh has rich tradition
of textiles, a big market and consumer base. It is
11. In which year is B.V. Nagarthana set to become home to hardworking weavers and a skilled
India's first female Chief justice ? workforce. The setting up of the PM MITRA Mega
(A) 2030 (B) 2027 Textiles Park across Lucknow and Hardoi districts
(C) 2024 (D) 2025 will greatly benefit UP".
"Today is a very important day for all my brothers
12. India works on three tiers of the Judicial system- and sisters of Uttar Pradesh. PM MITRA Park is
The Supreme Court, the High Court and going to be launched in Lucknow and Hardoi. My
Subordinate Courts. Which article talks about the best wishes to all of you on this occasion". he
establishment and constitution of Supreme Court? added.
(A) Article 123 (B) Article 128 "Spread over 1,000 acres, these PM MITRA parks
(C) Article 121 (D) Article 124 are going to give impetus to the local economy as
well as bring many new employment opportunities.
13. Who was the first judge against whom The textile sector of the country is also going to get
impeachment proceedings were initiated in new strength from them", PM further tweeted.
independent India? While speaking during a program of launch and
(A) V. Ramaswami J signing of Memorandum of Understanding (MoU)
(B) Soumitra Sen J of Integrated Textile Park in Lucknow on Tuesday
(C) C.V. Nagarjuna Reddy J along with state chief minister Yogi Adityanath,
(D) Dipak Misra CJI Union Commerce and Textiles minister Piyush
Goyal said that these parks will help generate
employment opportunities for the skilled manpower
of Uttar Pradesh.
[14]
While speaking during a program of launch and PASSAGE V
signing of Memorandum of Understanding (MoU) Addressing the opening session of the FIPIC-3
of Integrated Textile Park in Lucknow on Tuesday summit. Mr. Modi spoke in favour of a free and
along with state chief minister Yogi Adityanath, open Indo-Pacific region. He focused on India‟s
Union Commerce and Textiles minister Piyush commitment to assisting the development goals of
Goyal said that these parks will help generate the member countries of FIPIC and said New Delhi
employment opportunities for the skilled manpower will continue to help the Pacific island states in
of Uttar Pradesh. “every possible” way. “This time we are meeting
after a long gap. In the meanwhile, the world dealt
16. In which of the below following places is the with the COVID pandemic and several other global
India's first PM MITRA park situated? challenges. Climate change, natural calamities,
(A) Virudhunagar (B) Kamakshipalya poverty and famine already existed but now new
(C) Gorakhpur (D) Wayanad challenges are emerging as supply chains of food,
fuel fertilizer and pharma are facing hurdles. Those
17. Which of the below facts are true about Public we thought were reliable, we came to know they
Private Partnership? were not standing with us. In this time of difficulty,
(A) PPP is an arrangement between government the old saying that a friend in need is a friend in
and private sector for the provision of public deed has been proved. From vaccines to medicines,
assets and/or public services. Public-private wheat and sugar, India has supplied the countries
partnerships allow large-scale government that required these items. For me, you are large
projects, such as roads, bridges, or hospitals, to ocean countries and not small island nations. Your
be completed with private funding. ocean connects India with you,” said PM Modi.
(B) In this type of partnership, investments are The previous two meetings of the India-Pacific
undertaken by the government sector entity, Islands Cooperation were held in November 2014
for a specified period of time. in Fiji and August 2015 in Jaipur. Mr. Modi arrived
(C) The risk associated with the project is taken up on May 21 after completing his engagements at the
by the Government Entity G7 summit in Hiroshima where India was a guest
(D) All of the above facts are False country. Welcoming the guest, Prime Minister
James Marape said the members of the FIPIC have
18. What is the target year by which the parks will be suffered because of the high tariff on energy and
set up under PM-MITRA? food. “We suffered as a result of the big nations at
(A) 2026-2027 (B) 2029-2030 play in terms of geopolitics and power struggle,”
(C) 2032-2035 (D) 2039-2041 said Mr. Marape who is the co-chair of the FIPIC-3
Summit. “We are victims of global power play and
19. Which of the below following facts are true about we want you to be an advocate for us and sit in
Textile Sector in India? those meetings,” he said, urging India to serve as
(A) This sector is the 2nd largest provider of the voice of the Global South in the G-7 and the G-
employment in India, after agriculture. 20. Mr. Marape said the small size of the island
(B) India is the largest exporter of textile and nations of the Pacific Ocean region should not
apparel in the world overshadow the fact that the islands have large
(C) Both A and B stands true space in the ocean. Mr. Modi assured the FIPIC
(D) Both A and B stands false. members of India‟s support in the field of
sustainable development, science and technology
20. Who is the current Minister of Textiles? and space application.
(A) Rajeev Chandrashekar
(B) Piyush Goyal 21. How many Pacific Island Countries (PICs) are part
(C) Mansukh Mandaviya of the Forum for India-Pacific Islands Cooperation
(D) Dharmendra Pradhan (FIPIC)?
(A) 10 (B) 12
(C) 15 (D) 14

[15]
22. What is the objective of the Forum for India-Pacific was the tendency to alter (higher or lower) the
Islands Cooperation (FIPIC)? submission as per the entities‟ trading units‟
(A) To promote cultural and biological diversity in derivative positions to acquire more profits.
the Pacific region
Derivates refer to financial contracts whose value is
(B) To enhance India's relations with PICs in
various fields related to a specific indicator, commodity or
(C) To establish military alliances with Pacific financial instrument.
Island nations Indian Banks‟ Association (IBA) told The Hindu
(D) To compete with major powers like the US, that there were many products linked to LIBOR
Russia, and China in the Pacific region which had to be redesigned with an alternate
reference rate (ARR) as the base. Two working
23. Which country hosted the first FIPIC summit?
groups constituted by the association, receiving
(A) Fiji
(B) Papua New Guinea guidance from the RBI, helped develop the same.
(C) India Further, another set of challenges were related to
(D) Cook Islands technology and legal aspects (such as handling of
legacy contracts, modification of contracts with the
24. What is the total annual trade value between India counter parties/interbank as well as borrowers).
and Pacific Island countries? IBA notes the challenges were addressed “very
(A) USD 762million
effectively”. As for the essential systemic and
(B) USD 660 million
(C) USD 570 million technical changes, the association informed, “Banks
(D) USD 490 million have to identify the products that are linked to
Libor and the total exposure. Intimation to
25. Which Pacific Island country is India's biggest trade customers about the transition, insertion of fallback
partner in terms of value? clause in the contracts, assessment of the impact on
(A) Papua New Guinea
their P&L, changes in the technology platform etc.
(B) Fiji
(C) Cook Islands are needed to facilitate the transition,” adding, “We
(D) Nauru understand the banks are in readiness.” For
perspective, fullback clauses refer to an agreement
PASSAGE VI for revised considerations when the reference rate is
Every business day before 11 a.m. (London time), no more published – important for transparency and
banks on the LIBOR panel make their submissions consistency.
to news and financial data company, Thomson
Reuters. The panel consists of commercial bankers 26. What is LIBOR?
such as J.P. Morgan Chase (London branch), (A) A benchmark interest rate
Lloyds Bank, Bank of America (London branch), (B) A financial data company
Royal Bank of Canada and UBS AG, among others. (C) A news company
(D) A type of derivative
Following the submission, the contributed rates are
ranked. Extreme quartiles, on the top and bottom, 27. What is the alternative to LIBOR?
are excluded and the middle quartiles are averaged (A) MMIFOR (B) MIFOR
to derive the LIBOR. The idea is to be as close to (C) SOFR (D) SOIFR
the median as possible. Therefore, it would not be
particularly useful to impress upon potential and 28. Why is RBI moving away from LIBOR?
current customers the various disadvantages in (A) Due to concerns over its reliability and
obtaining funds. The phenomenon was particularly integrity
(B) Due to its heavy reliance on banks
on display during the 2008 financial crisis when
(C) Due to its susceptibility to market
submissions were artificially lowered (amid the manipulation
crisis). In 2012, Barclays admitted to the (D) All of the above
misconduct and agreed to pay $160 million in
penalties to the U.S. Dept of Justice. The Wall 29. What does MMIFOR incorporate?
Street Journal too had studied in May 2008 that (A) Adjusted MIFOR rates
several panelists were paying “significantly lower (B) Adjusted SOFR rates
borrowing costs” than what other market measures (C) U.S. Treasury securities
were suggesting. Another observed phenomenon (D) Bloomberg Index Services

[16]
30. What challenges does transitioning from LIBOR to
an ARR pose?
(A) Legal aspects
(B) Technology aspects
(C) Dealing with existing contracts
(D) All of the above

[17]
Legal Reasoning
PASSAGE I manner for the public. Section 4(4) outlines factors
Article 19(1)(a) of the Indian Constitution grants to consider during information dissemination, such
every citizen the right to free speech and as cost-effectiveness, using the local language, and
expression. This right encompasses both the selecting the most effective communication method
dissemination and receipt of information, as being for a particular area.
adequately informed is essential for forming Section 5 of the Act pertains to the designation of
informed opinions. Therefore, the right to seek and Public Information Officers (PIOs). Public
acquire information is an integral aspect of the authorities must appoint Central and State PIOs
fundamental right protected by Article 19(1)(a). within 100 days of the Act's enactment. These
The Right to Information (RTI) Act of 2005 aims to officers have the responsibility to provide
achieve several objectives: information requested under the Act. Additionally,
Establishing a practical framework: The RTI Act Section 5(2) calls for the designation of Assistant
provides a framework that enables citizens to access PIOs at lower administrative levels to receive
information held by public authorities. applications and appeals and forward them to the
Promoting transparency and accountability: The appropriate authorities.
Act aims to enhance transparency and In summary, the RTI Act of 2005 was enacted to
accountability in the functioning of government ensure access to information, promote transparency
bodies and related entities. and accountability, combat corruption, and foster an
Establishing Information Commissions: The Act informed citizenry.
provides for the creation of Information
Commissions at both the state and national levels. 1. A citizen filed an application seeking information
These commissions are responsible for executing about the utilization of public funds by a
the functions and exercising the powers outlined in government department. The Public Information
Officer (PIO) denied the request, stating that it
the Act.
would adversely affect the economic interests of the
Developing an informed citizenry: By granting country. Was the PIO's decision justified under the
access to information, the Act seeks to foster an RTI Act?
informed citizenry capable of participating in (A) Yes, because disclosure of such information
democratic processes. could harm the economic interests of the
Combating corruption: The Act aims to curb country.
corruption by facilitating access to information and (B) Yes, because economic interests take
precedence over the right to information.
promoting accountability within public authorities.
(C) No, because the citizen has the right to access
Balancing conflicting interests: The Act delineates information about the utilization of public
exemptions to the disclosure of information in cases funds.
where such disclosure may conflict with other (D) No, because the PIO is obligated to disclose
public interests. It seeks to harmonize these information unless it falls within the specified
conflicting interests while upholding the exemptions.
paramountcy of democratic ideals.
2. A public authority receives an application seeking
Section 4(1) of the Act imposes specific obligations
information about an ongoing investigation. The
on public authorities, including the maintenance of authority denies the request, claiming that it could
records. Public authorities are required to organize impede the investigation process. Is the denial
and index their records and, where feasible, justified under the RTI Act?
computerize them for ease of access. They must (A) Yes, because the disclosure of information
also publish certain particulars within 120 days of about ongoing investigations can hinder the
investigative process.
the Act's enactment.
(B) Yes, because protecting the integrity of
Section 4(2) mandates that public authorities ongoing investigations is of higher importance
proactively provide information to the public than the right to information.
through various means of communication at regular (C) No, because citizens have the right to access
intervals. information unless it falls within the specified
Sections 4(3) and 4(4) focus on the dissemination exemptions.
of information. Section 4(3) emphasizes the wide (D) No, because the RTI Act does not provide
exemptions for ongoing investigations.
dissemination of information in an easily accessible
[18]
PASSAGE II
3. A public authority failed to publish certain The Right to Freedom of Religion is guaranteed to
particulars within the prescribed timeframe of 120 all Indians by the Constitution under Articles 25 to
days. Can a citizen file a complaint against the
28. This topic, which is intertwined with the
authority under the RTI Act?
(A) Yes, because the public authority has a legal concept of secularism is very important.
obligation to publish the particulars within the Right to Freedom of Religion
specified timeframe. The Constitution of India guarantees the right to
(B) Yes, because the citizen has the right to hold freedom of religion to not only individuals but also
the public authority accountable for non- religious groups in India. This is enshrined in
compliance with the RTI Act. Articles 25 to 30.
(C) No, because the RTI Act does not provide for
Article 25 (Freedom of conscience and free
complaints against public authorities.
(D) No, because the prescribed timeframe of 120 profession, practice, and propagation of religion)
days is not mandatory but only a suggested Article 25 guarantees the freedom of conscience,
guideline. the freedom to profess, practice, and propagate
religion to all citizens.
4. A citizen submits an application seeking The above-mentioned freedoms are subject to
information from a public authority but does not public order, health, and morality.
receive a response within the specified timeframe.
This article also gives a provision that the State can
Can the citizen file an appeal under the RTI Act?
(A) Yes, because the citizen has the right to appeal make laws:
if the public authority fails to respond within That regulates and restricts any financial, economic,
the specified timeframe. political, or other secular activity associated with
(B) Yes, because the citizen has the right to access any religious practice.
information and seek remedies for non- That provides for the social welfare and reform or
compliance with the Act.
opening up of Hindu religious institutions of a
(C) No, because the RTI Act does not provide for
the right to appeal in case of non-response public character to all sections and classes of
from public authorities. Hindus. Under this provision, Hindus are construed
(D) No, because the specified timeframe for as including the people professing the Sikh, Jain, or
response is not legally binding and only serves Buddhist religions, and Hindu institutions shall also
as a guideline. be construed accordingly.
People of the Sikh faith wearing & carrying the
5. A public authority receives a request for kirpan shall be considered as included in the
information but refuses to provide it, citing the lack profession of the Sikh religion.
of resources to disseminate the information Article 26 (Freedom to manage religious affairs)
effectively. Is the public authority's refusal justified This Article provides that every religious
under the RTI Act? denomination has the following rights, subject to
(A) Yes, because the public authority is not morality, health, and public order.
obliged to disseminate information if it lacks The right to form and maintain institutions for
the necessary resources. religious and charitable intents.
(B) Yes, because the cost-effectiveness and The right to manage its own affairs in the matter of
effectiveness of information dissemination religion.
methods are important considerations. The right to acquire the immovable and movable
(C) No, because the RTI Act mandates public property.
authorities to disseminate information in an The right to administer such property according to
easily accessible manner. the law.
(D) No, because the public authority must allocate Article 27 (Freedom as to payment of taxes for
resources to ensure effective information promotion of any particular religion)
dissemination. According to Article 27 of the Constitution, there
can be no taxes, the proceeds of which are directly
used for the promotion and/or maintenance of any
particular religion/religious denomination.

[19]
Article 28 (Freedom as to attendance at religious you can give in court defending the decision of state
instruction or religious worship in certain of Madhya Pradesh
educational institutions) (A) This law is Valid as sacrificing children is an
This article permits educational institutions that are immoral act and article 25 is subject to
maintained by religious groups to disseminate Abhijeet's morality
religious instruction. (B) This law is Valid as sacrificing children is an
This provides that no religious instruction shall be immoral act and article 25 is subject to Public
provided in State-run educational institutions. Health
Educational institutions administered by the State (C) This law is Valid as sacrificing children is an
but that were established under any endowment or immoral act and article 25 is subject to
trust which requires that religious instruction shall community health
be imparted in such institutions are exempt from the (D) All the above
above clause (that no religious instruction shall be
provided). 8. PCN community is a minority community in cest
Any person who attends any educational institution pengal state. Chief minister of the state hates them
recognized by the State or receiving State aid shall and stops them from dissipating religious
not be required to participate in any religious knowledge in educational institutions managed by
instruction that may be imparted in such institution, the community. Member of the PCN community
or also attend any religious worship in such want to file a case for violation of their
institutions unless he/she has given consent for the constitutional rights. You are their lawyer tell
same. In the case of minors, the guardians should which right is violated here?
have given consent for the same (A) religious rights under article 25 are violated
here as it permits educational institutions that
6. INIM is a religious organization in Madhya Pradesh are maintained by religious groups to
state in a country called Ifia which practices a disseminate religious instruction
practice of throwing Childrens in open fire as a part (B) religious rights under article 26 are violated
of their religious ceremony. Chief minister of here as it permits educational institutions that
Madhya Pradesh Abhijeet Singh Passes a law are maintained by religious groups to
prohibiting this practice citing children rights. disseminate religious instruction
Many communist organizations protest and (C) religious rights under article 27 are violated
challenge these laws in supreme court. You‟re the here as it permits educational institutions that
chief justice of Supreme court decide the case are maintained by religious groups to
(A) The law is invalid as it unjustifiably infringes disseminate religious instruction
upon the right to freedom of religion protected (D) religious rights under article 28 are violated
under article 25 state should stay away from here as it permits educational institutions that
religious matters are maintained by religious groups to
(B) This law is Valid as sacrificing children is an disseminate religious instruction
immoral act and article 25 is subject to
morality 9. Pota is a minority religious community in Punjab
(C) Abhijeet Singh is a fascist because he province of British Raj which practices drinking
unjustifiably infringes upon the right to blood of humans. Governer of Punjab Lord Promod
freedom of religion of INIM organization prohibits this citing Human rights. Members of the
(D) Both A and C religious community challenge the law, arguing that
it infringes upon their right to freedom of religion
7. INIM is a religious organization in Madhya Pradesh under Article 25. decide the case
state in a country called Ifia which they consume (A) This law is valid as religious freedoms under
Drugs as a part of their religious ceremony. Chief article 25 are subject to some restrictions
minister of Madhya Pradesh Abhijeet Singh Passes (B) This law is valid as religious freedoms under
a law prohibiting this citing health concerns. Some article 25 are subject to morality and this
5-year law students of Palsar University challenge practice is Immoral
this law. You‟re the counsel for the state of Madhya (C) This law is Invalid as religious freedoms under
Pradesh. Which among the following is the reason article 25 are not subject to any restrictions

[20]
(D) None of the above Section 12(5): As per sub-clause (5) of Section 12,
the Chief Information Commissioner and other
10. Pontie community of Madhya Pradesh wants to Information Commissioners should be the person of
purchase an immovable property but they were acknowledge superiority in public life and possess
stopped by a local MLA as he uses this property for wide knowledge and experience in the field of law,
urinating. Pontie community files a case for technology, social science, management, science,
violation of rights. You are CJM ria Khalifa decide mass media, journalism, governance, and
the case administration.
(A) There is violation of Article 26 (Freedom to Section 12(6): As per sub-clause (6) of Section 12,
manage religious affairs) which includes the Chief Information Commissioner and other
purchase of immovable property Information Commissioners shall not hold any
(B) There is violation of Article 25 (Freedom to office of profit or should not be the Member of
manage religious affairs) which includes Parliament or shall not be the Member of any
purchase of immovable property State/Union Territory Legislative Assembly, or
(C) There is violation of Article 28 (Freedom to shall not be pursuing any profession or carrying any
manage religious affairs) which includes business or shall not be connected with any political
purchase of immovable property party.
(D) Article 26 (Freedom to manage religious Section 18 of the Right to Information Act, 2005
affairs) which includes purchase of immovable talks about the powers and functions of the
property is subject to reasonable restriction and Information Commission.
this restriction is reasonable who has been unable to submit a request for
information to a Public Information Officer either
PASSAGE III at central or state level, either by the reason that no
Section 12 of the Right to Information Act, 2005 such officers have been appointed under this statute
deals with the constitution of a statutory body or the appointed officers refuse to accept his/her
known as the Central Information Commission. application;
According to this provision, the central government who has been refused to get access to the
shall constitute a body called the Central information requested under this statute;
Information Commission bypassing a notification who has not been receiving any response for the
in the Official Gazette.Section 12 of the Right to application he/she made for information and access
Information Act, 2005 is termed as the constitution to information with the limited time as per this
of the Central Information Commission. legislation;
Furthermore, the salient features of this section are who has been required to pay an unreasonable
as follows: amount of fee;
Section 12(1): This subsection empowers the who believes that the information provided by the
central government to constitute a body known as officers are false,
the Central Information Commission.
Section 12(2): As per sub-clause (2) of Section 12, 11. Mr. Sharma, a citizen, has been trying to submit a
the Central Information Commission should consist request for information to a Public Information
of the Chief Information Commissioner and such Officer (PIO) at the central level, but no such
officers have been appointed. Can Mr. Sharma seek
other Chief Information Commissioners not more
relief under the Right to Information Act, 2005?
than 10 as may be deemed necessary. (A) Yes, because the Act guarantees the right to
Section 12(3): As per sub-clause (3) of Section 12, information at both the central and state levels.
the Chief Information Commissioner and other (B) Yes, because the Act allows citizens to
Information Commissioners shall be appointed by approach the Central Information Commission
the President of Republic of India on the directly.
recommendation of a committee consisting of; the (C) No, because Mr. Sharma has not exhausted all
possible avenues to contact a PIO.
Prime Minister of India, as the Chairperson of the
(D) No, because the Act does not provide any
committee; the leader of the single largest group in remedy in case no PIOs are appointed.
opposition of the Government of India in Lok
Sabha; the Union Cabinet member shall be
nominated by the Prime Minister of India.

[21]
12. Ms. Singh applied for access to certain information (B) Yes, because the Act empowers the Chief
under the Right to Information Act, 2005 but was Information Commissioner to investigate false
denied by the appointed officers. Can Ms. Singh information.
seek redress under the Act? (C) No, because Mr. Gupta should have provided
(A) Yes, because the Act grants citizens the right evidence to prove the falsity of the
to access information upon request. information.
(B) Yes, because the Act empowers the Chief (D) No, because the Act does not hold officers
Information Commissioner to enforce access to accountable for the accuracy of the
information. information they provide.
(C) No, because Ms. Singh did not provide
sufficient grounds for requesting the PASSAGE IV
information. Arbitration is usually faster, simpler, more efficient,
(D) No, because the Act does not provide any and more flexible for scheduling than litigation. It
recourse for denial of access to information. requires less documentation to file and obtaining a
decision is quicker than in the context of the
13. Mr. Patel submitted an application for information judicial process. Also, if the subject of the dispute
but did not receive any response within the is technical (for example - about a patent) the
specified time frame mentioned in the legislation. parties can select an arbitrator who has technical
Can Mr. Patel take action under the Right to knowledge in that field rather than a judge who
Information Act, 2005? may not be familiar with the issues.
(A) Yes, because the Act guarantees a timely Appointment of Arbitrators (Section 11): - The Act
response to information requests. allows full freedom to the parties in the matter of
(B) Yes, because the Act allows citizens to file a appointment of arbitrators, including by agreement,
complaint with the Central Information selecting from a list of arbitrators, or via a process
Commission. of elimination. However, if there is a failure of the
(C) No, because Mr. Patel should have followed up parties‟ agreed mechanism for an appointment, the
with the designated officer for a response. Chief Justice of a High Court (in the case of a
(D) No, because the Act does not impose any domestic arbitration) or the Chief Justice of the
obligation on the officers to respond within a Supreme Court of India (in the case of international
specific time frame. commercial arbitration) may be approached for this
purpose. This is the first instance in which the Act
14. Mr. Khan was asked to pay an exorbitant fee for envisages recourse to a court concerning an
accessing the requested information. Can Mr. Khan arbitration proceeding.
challenge the fee charged under the Right to Interim Relief (Section 9) & (Section 17): - A party
Information Act, 2005? that seeks interim measures has essentially two
(A) Yes, because the Act prohibits unreasonable avenues open to it. It can approach the court, or it
fee requirements for accessing information. can approach the arbitral tribunal. Section9 enables
(B) Yes, because the Act empowers the Chief a party to approach a competent court before or
Information Commissioner to review and during the arbitral proceedings or even after the
modify fees. award is pronounced, but before it is enforced.
(C) No, because Mr. Khan should have negotiated Section 17 enables the arbitral tribunal to take
the fee with the appointed officers. interim measures in the subject matter of the
(D) No, because the Act allows officers to dispute at the request of the parties.
determine the fee based on their discretion. Finality of an arbitral award (Section 34): - An
arbitral award is regarded as final and binding on
15. Mr. Gupta believes that the information provided both the parties and could be enforced like a court
by the officers is false. Can Mr. Gupta seek redress decree. The recourse available against an arbitral
under the Right to Information Act, 2005? award is by moving an application for setting aside
(A) Yes, because the Act guarantees the accuracy an arbitral under Section 34 on the grounds as
and authenticity of the provided information. mentioned in the provisions, which include but not
limited to when an Arbitrator is biased, or the

[22]
award rendered is against the public policy or the 18. Party A and Party B include an arbitration clause in
Arbitrator has exceeded its jurisdiction. their contract, stating that any disputes will be
Appeals (Section 37): - An appeal shall lie against resolved through arbitration. However, they fail to
agree on the number of arbitrators involved. What
the decision of the Court passed in Section 8,
is the likely consequence?
Section 9 and section 34 petitions. (A) The Chief Justice of the Supreme Court will
Arbitration clauses are increasingly finding their appoint a single arbitrator to resolve the
way into commercial contracts and easy and viable dispute.
modes of settling disputes to arbitration. However, (B) The parties will have to mutually agree on the
the parties to the Contract have to be conscious at number of arbitrators before arbitration can
the time of drafting the arbitration clause, since it proceed.
(C) The arbitration clause will be deemed invalid,
governs the future proceedings in case of disputes.
and the dispute will be automatically resolved
The governing law regarding arbitration
through litigation.
proceedings, substantive rights of the parties to
(D) The default number of arbitrators specified in
arbitration, the seat and venue of arbitration, etc.
the arbitration law will be applicable.
has to be clearly defined to avoid unnecessary
dispute later on.
19. Party A and Party B include an arbitration clause in
their contract, stating that any disputes will be
16. In a commercial contract, Party A and Party B
resolved through arbitration. Party A later claims
include an arbitration clause stating that any
that the arbitral award is against public policy and
disputes arising out of the contract will be resolved
seeks to set it aside under Section 34. Can Party A
through arbitration. However, they fail to specify
do so?
the governing law regarding arbitration
(A) Yes, if Party A can prove that the arbitrator
proceedings. In the event of a dispute, what is the
was biased.
likely consequence?
(B) Yes, if Party A can demonstrate that the
(A) The arbitration clause will be deemed invalid,
arbitration proceedings violated the governing
and the dispute will be automatically resolved
law.
through litigation.
(C) No, because an arbitral award is regarded as
(B) The parties will have to mutually agree on the
final and binding, and the grounds for setting it
governing law before arbitration can proceed.
aside are limited.
(C) The Chief Justice of the Supreme Court will
(D) No, because the Chief Justice of the Supreme
appoint an arbitrator to determine the
Court has exclusive jurisdiction over setting
governing law
aside arbitral awards.
(D) The court will determine the governing law
based on the applicable legal provisions
20. Party A and Party B include an arbitration clause in
their contract, specifying that any disputes will be
17. Party A initiates arbitral proceedings against Party
resolved through arbitration. Party A initiates
B, seeking interim measures to protect its interests.
arbitration proceedings but fails to comply with the
However, the arbitral tribunal refuses to grant the
agreed-upon process of elimination for arbitrator
requested interim measures. Can Party A approach
selection. What can Party A do?
the court for interim relief?
(A) Party A can approach the Chief Justice of the
(A) Yes, because Section 17 allows the arbitral
Supreme Court for the appointment of an
tribunal to take interim measures at the request
arbitrator.
of the parties.
(B) Party A can request the arbitral tribunal to
(B) Yes, if Party A can demonstrate that the
arbitral tribunal's decision is biased. select an arbitrator from a list of arbitrators.
(C) No, because the court's jurisdiction is limited (C) Party A can initiate litigation proceedings
to enforcing arbitral awards, not granting instead of arbitration.
interim relief. (D) Party A can choose an arbitrator unilaterally
(D) No, because Party A has already exhausted all without following the agreed-upon process.
available avenues for seeking interim
measures.

[23]
PASSAGE V (1) It is an executive power vested in the officer so
Section 144 of the Code of Criminal Procedure is empowered.
the most utilitarian and frequently used section. (2) There must exist sufficient ground for
Empowered Magistrates are conferred with great Proceeding
powers under this particular section basically to (3) Immediate prevention or speedy remedy is
issue orders in urgent cases of either nuisance or desirable, and
apprehended danger. Here, nuisance or (4) An order, in writing, should be passed stating
apprehended danger in a way signifies the the material facts and be served the person
situations which either create or contains the
potential to create an atmosphere of unrest among 21. Two companies were competitors in business. One
people in a society or to cause danger to public of the executives of Company A provided evidence
health, peace, or safety. Authorities are empowered against the illegal activities of Company B to the
under this section to bar the assembly of 5 or more police. Based on this evidence, several premises of
people in a given jurisdiction or a specific area Company B were sealed. In response, the CEO of
which demands the urgency to impose the Company B threatened to destroy the buildings of
particular section. And the very urgency of the case Company A as punishment for the informant. They
even demands to lay aside the formalities and stated that they would accomplish this by using a
preliminaries usually required for the making of an wrecking ball machine during the night.
order. An order under this section can only be Subsequently, a sub-divisional magistrate issued an
passed by specified classes of Magistrates when in order to submit all the keys of the wrecking ball
their opinion there is sufficient ground for machine to the nearest court.Decide whether the
proceedings under the section and immediate order is valid or not
prevention or speedy remedy is desirable Section (A) The order is valid because it aims to prevent
144 CrPC this provision is now very familiar to potential damage and maintain public safety.
people, thanks to its widespread invocation by (B) The order is invalid as it infringes upon the
police forces across the country. Section 144 gives rights of Company B without due process.
power to a District Magistrate, a sub-divisional (C) The order is invalid as it exceeds the authority
Magistrate, or any other Executive Magistrate of the sub-divisional magistrate.
empowered by the State Government for the same, (D) The order is invalid as the sub-divisional
to issue an proportinate order to an individual or magistrate cannot pass an order on the grounds
the general public particular place or area to abstain mentioned.
from a certain act or "to take certain order with
respect to certain property in possession or under 22. An Executive Magistrate issues an order prohibiting
his management". The order can be passed even ex- the distribution of a particular newspaper within a
parte, if need be specific region due to alleged misinformation
As per the Section, the order can be passed only "il without any sufficient proof. Evaluate the validity
such Magistrate considers, that the direction is of this order.
likely to prevent: (A) The order is valid as it prevents the
obstruction, annoyance or injury to any person dissemination of potentially harmful
lawfully employed. misinformation.
danger to human life, health or safety. (B) The order is invalid as it infringes upon the
disturbance of the public tranquillity, or a riot or freedom of the press without sufficient legal
affray. basis.
In Babulal Parate v State of Maharashtra (1961) (C) The validity of the order depends on the
case, the SC held that use of this power should be specific evidence provided to establish the
basedsufficient materials which show that misinformation and its potential harm.
immediate prevention of certain acts is necessary to (D) The order is invalid as the Executive
preserve public "The test laid down in the Section is Magistrate does not have the authority to issue
not mere likelihood' or tendency." As explained in orders under section 144 without sufficient
In Re: Ramlila Incident Case (2012), the necessary evidence
prerequisites of the order under Section 144

[24]
23. A sub-divisional Magistrate issues an order link between the rise in criminal activities and
restricting the freedom of speech and expression in the need for a curfew.
a particular area to prevent the spread of potentially (D) The order is invalid as the Commissioner does
inflammatory content. Assess the validity of this not have the authority to issue orders imposing
order. a curfew in the city.
(A) The order is valid as it aims to maintain public
order and prevent potential harm. PASSAGE VI
(B) The order is invalid as it infringes upon the According to the definition given by Halsbury, the
fundamental right to freedom of speech term “indemnity” is a contract that expressly or
without sufficient justification. impliedly protects a person who entered into a
(C) The validity of the order depends on the contract or is about to enter from any losses,
specific evidence provided to establish a direct irrespective of the fact that those losses were due to
link between the content and potential harm to the actions of a third party.
public order. For the purpose of a contract of indemnity, the
(D) The order is invalid as the sub-divisional following conditions must be satisfied:
Magistrate does not have the authority to issue There must be two parties.
orders restricting freedom of speech and One of the parties must promise the other to pay for
expression. the loss incurred.
The contract may be expressed or implied.
24. The District Magistrate (DM) of Dholakpur issued It must satisfy the essentials of a valid contract.
an order in response to information received by Conditions for the contract of indemnity:
Chota Bheem regarding a potential bomb at the As mentioned above, there must essentially be two
King's palace. The order states that only "officers parties in a contract of indemnity: the indemnity
above the rank of inspector or individuals holder and the indemnifier. Moreover, no individual
authorized by them are allowed within a 5 km can enter into a contract with themselves, and the
radius of the palace". Evaluate the validity of this minimum requirement for any contract to be legally
order. valid is for two parties.
(A) The order is valid as it is necessary to ensure Promise
the safety and security of the King's palace in A contract of indemnity is one in which one party
response to the potential bomb threat. promises to protect the other party from harm
(B) The order is invalid as the magistrate didn't brought on by the actions of the other party.
state the full reason of promulgation of order One party must present a condition to another party,
(C) The order is invalid as the reason provided by and the other party must accept it. Acceptance
Chota Bheem for issuing the order is occurs when another party accepts the offer on the
inadequate and lacks proper evidence. same terms. After accepting the offer, it becomes a
(D) The order is invalid as the DM of Dholakpur promise. The party that made the promise is now
does not have the authority to issue such known as the promisor, and the person who
restrictive orders in response to potential accepted it is now known as the promisee.
threats. It is an important part of the contract of indemnity
that “the promise must be made by the promisor to
25. The Commissioner of a city issues an order pay the losses of the promisee.” A contract of
imposing a curfew from 10:00 PM to 5:00 AM in indemnity is one in which one party promises to
response to a rise in criminal activities during protect the other party from harm brought on by the
nighttime. Assess the validity of this order. actions of the other party.
(A) The order is valid as it aims to ensure public Expressed or implied
safety and reduce criminal activities during the As stated above, a contract for indemnity may be
specified hours. expressed or implied. In other words, parties may
(B) The order is invalid as it infringes upon the directly impose their own conditions in such a
freedom of movement of individuals without contract. The nature of circumstances may also
sufficient legal justification. create indemnity obligations impliedly. Express
(C) The validity of the order depends on the contracts are those that are created orally or in
specific evidence provided to establish a direct

[25]
writing, whereas implied contracts are those that are (B) Yes, only if R can demonstrate that the loss
made as a result of the conduct of the parties. suffered exceeds any reasonable limit that
There must be a loss incurred could have been specified.
The condition of the contract of indemnity is that (C) No, unless R and S had explicitly agreed to
“the loss must be incurred by the promisee.” The unlimited indemnification in their contract.
promisor is not required to make any payments if (D) Yes, because the absence of a specified limit
the promisee suffers no loss. allows R to claim unlimited indemnification.
Lawful object and consideration
A contract for indemnity can only be executed for a 29. X, as the indemnity holder, suffers a loss due to
valid purpose and a lawful consideration. A their own negligent actions. Can X demand
contract of indemnity cannot be construed as a indemnification from Y, the indemnifier?
contract to engage in unlawful behaviour or (A) No, because a contract of indemnity does not
conduct that is against public policy. cover losses caused by the negligence of the
indemnity holder.
26. T and U enter into a contract of indemnity without (B) No, unless X and Y had explicitly included
any consideration exchanged between them. Is the coverage for losses resulting from the
contract valid? indemnity holder's negligence in the contract.
(A) Yes, because a contract of indemnity does not (C) Yes, because Y, as the indemnifier, is
require consideration to be legally enforceable. responsible for any losses incurred by X,
(B) Yes, only if T and U can prove that there was a regardless of the cause.
valid intention to enter into a contractual (D) Yes, only if X can prove that Y explicitly
agreement. agreed to cover losses resulting from X's own
(C) No, because a contract of indemnity must negligence.
include a lawful consideration for it to be
valid. 30. X enters into an implied contract of indemnity with
(D) No, unless T and U can demonstrate that the Y. X incurs a loss due to the actions of Z, a third
absence of consideration was mutually agreed party. Can X demand indemnification from Y?
upon. (A) Yes, because an implied contract of indemnity
can extend to cover losses caused by third
27. R and S enter into a contract of indemnity, but the parties.
purpose of the contract involves selling drugs. Is (B) Yes, only if X can prove that Y had prior
the contract of indemnity enforceable? knowledge of the potential loss caused by Z.
(A) Yes, because a contract of indemnity is valid (C) No, because an implied contract of indemnity
as long as both parties have mutual consent. only applies to losses caused directly by the
(B) Yes, only if R and S can demonstrate that the indemnifier.
illegal activities were necessary for their (D) No, unless X and Y explicitly agreed to
mutual benefit. include third-party losses in their contract.
(C) No, because a contract of indemnity cannot be
construed as a contract to engage in unlawful
behavior.
(D) No, unless R and S can prove that the illegal
activities were conducted within the
boundaries of the law.

28. R and S enter into a contract of indemnity, but the


contract does not specify any limit on the indemnity
amount. Can R claim unlimited indemnification
from S?
(A) No, because an indemnity amount must be
reasonable and within the scope of the
contract's terms.

[26]
Quantitative Techniques
PASSAGE I PASSAGE II
Directions: Study the following data carefully Directions: Study the following data carefully and
and answer the questions accordingly. answer the questions accordingly.
The given pie chart shows the degree distribution of The given Bar graph shows the number of three
different rupee notes i.e. 500, 200 and 100 with five
mobile phones produced in five different days
different shopkeepers (A, B, C, D and E) on Sunday.
namely Monday, Tuesday, Wednesday, Thursday
and Friday.

6. The amount corresponding to the number of 200


rupee notes with shopkeeper B is what percentage
1. If out of the total number of mobile phones
of the amount corresponding to the number of 500
produced on Wednesday, 20% of the mobile phones
rupee notes with shopkeepers A and E together?
are defective, then find the total number of mobile
(A) 35.23% (B) 44.56%
phones which are non-defective on Wednesday.
(C) 58.46% (D) 82.31 %
(A) 150 (B) 480
(E) None of these
(C) 560 (D) 600
(E) None of these
7. If the shopkeeper C wants to buy a smartphone
and clothes worth Rs.9200 and Rs.4000 and then
2. Find the ratio between the total number of mobile
find 15% of the amount is left with shopkeeper C?
phones produced on Thursday to the total number
of mobile phones produced on Friday (A) Rs.1950 (B) Rs.1550
(A) 2 : 5 (B) 1 : 3
(C) 4 : 3 (D) 1 : 1 (C) Rs.2210 (D) Rs.1640
(E) None of these
(E) None of these
3. Find the sum of the total number of Mobile phones
produced on Wednesday and Thursday. 8. Find the ratio of the number of 200 and 100 rupee
(A) 1050 (B) 1200 notes with shopkeeper E to the number of 500
(C) 1100 (D) 1250 rupee notes with shopkeepers B and C together?
(E) None of these (A) 9:4 (B) 5:8
(C) 14:5 (D) 9:5
4. Find the average number of mobile phones (E) None of these
produced on Tuesday, Wednesday and Thursday.
(A) 200 (B) 250 9. If the total number of 500 and 100 rupee notes with
(C) 450 (D) 150 shopkeeper F is 28.56% more than that of
(E) None of these shopkeeper C and the ratio of the number of 200
rupee notes with shopkeepers F and D is 5:4. Then
5. The number of mobile phones produced on Monday find the average number of 200, 100 and 50 rupee
is what percent of the the number of mobile phones notes with shopkeeper F?
produced on Wednesday? (A) 60 (B) 40
(A) 50% (B) 40% (C) 80 (D) 20
(C) 60% (D) 70% (E) None of these
(E) None of these
[27]
10. The total amount with shopkeeper D from 500
rupee notes is how much more/less than the total
amount with shopkeeper B from 100 rupee notes?
(A) Rs.4200 (B) Rs.8500
(C) Rs.9200 (D) Rs.8400
(E) None of these

[28]
Answer Key
Section I 6. (B) 12. (D) 13. (B)
1. (B) 7. (A) 13. (A) 14. (A)
2. (A) 8. (A) 14. (C) 15. (C)
3. (B) 9. (A) 15. (D) 16. (B)
4. (B) 10. (C) 16. (A) 17. (B)
5. (D) 11. (B) 17. (A) 18. (B)
6. (A) 12. (C) 18. (A) 19. (C)
7. (B) 13. (A) 19. (A) 20. (B)
8. (C) 14. (C) 20. (B) 21. (D)
9. (D) 15. (B) 21. (D) 22. (D)
10. (D) 16. (C) 22. (B) 23. (D)
11. (A) 17. (C) 23. (A) 24. (B)
12. (A) 18. (B) 24. (C) 25. (D)
13. (A) 19. (B) 25. (A) 26. (C)
14. (A) 20. (C) 26. (A) 27. (C)
15. (D) 21. (D) 27. (C) 28. (C)
16. (B) 22. (B) 28. (D) 29. (A)
17. (B) 23. (D) 29. (B) 30. (C)
18. (C) 24. (B) 30. (D)
19. (C) 25. (A) Section V
20. (C) Section IV 1. (D)
21. (A) Section III 1. (D) 2. (B)
22. (B) 1. (A) 2. (A) 3. (A)
23. (B) 2. (A) 3. (B) 4. (C)
24. (A) 3. (B) 4. (A) 5. (C)
25. (C) 4. (B) 5. (C) 6. (C)
5. (B) 6. (B) 7. (A)
Section II 6. (A) 7. (B) 8. (C)
1. (C) 7. (C) 8. (D) 9. (B)
2. (D) 8. (B) 9. (B) 10. (D)
3. (A) 9. (D) 10. (A)
4. (C) 10. (C) 11. (D)
5. (C) 11. (B) 12. (A)

[29]
Hints & Solutions
Section I beautiful young woman. To find an antonym, we are
PASSAGE I looking for a word that contrasts with the
1. Correct Option B characteristics of a nymph. Let's examine the
(H & S) options:
The correct answer is b) Diogenes' fascination with A) Sprite: A sprite is also a mythological creature
the causes of masturbation is merely a crude joke associated with nature, often depicted as a small,
without any deeper significance. This can be inferred mischievous spirit. This choice does not serve as an
from the passage where it states that Diogenes' antonym to "nymph."
fascination with the causes of masturbation offers B) Human: Humans are not mythological creatures
more than just a crude joke. The passage suggests and do not possess the same attributes as nymphs.
that there is a deeper significance to Diogenes' views This option could potentially serve as an antonym to
on the subject. "nymph" as it represents the opposite of a mythical
being.
2. Correct Option A C) Specter: A specter refers to a ghostly or haunting
(H & S) figure, which is unrelated to the characteristics of a
The tone of the passage can be inferred from the nymph. This choice does not serve as an antonym to
content and language used. The passage contains "nymph."
elements of humor, as seen in the crude jokes and D) Dryad: A dryad is another type of mythological
references to Diogenes' public demonstrations. It creature associated with nature, specifically a tree
also touches upon ecological concerns and Diogenes' nymph. This option does not serve as an antonym to
philosophical perspective on autonomy and self- "nymph."
sufficiency. While there is an element of satire in the Based on the analysis, the suitable antonym to
passage, it is not the dominant tone. Therefore, the "nymph" would be option B) Human, as it represents
most appropriate answer would be: HUMOROUS the opposite of a mythological creature.

3. Correct Option B 5. Correct Option D


(H & S) (H & S)
The passage discusses the Cynic philosopher The correct answer is D) According to Dio
Diogenes of Sinope's fascination with masturbation, Chrysostom, Diogenes' fascination with the causes of
masturbation was solely for crude jokes. The passage
its ecological significance, and its potential impact
suggests that Diogenes saw masturbation as a form
on the world. Based on this information, the most of autonomy and self-sufficiency, believing it could
suitable title for the passage would be: The lead to a better world.

Ecological Gesture of Diogenes: Masturbation as


PASSAGE II
Autonomy
6. Correct Option A
(H & S)
4. Correct Option B The correct answer is (a) India's nuclear tests in 1974
and 1998 caused economic sanctions that severely
(H & S)
impacted its economy.
The word "nymph" refers to a mythological creature
associated with nature and typically depicted as a
[30]
Explanation: The passage mentions that the US-led Nuclear power development: This option refers to
economic sanctions following the tests did not have the growth or advancement of a country's nuclear
the terrible repercussions they once may have since power capabilities.
the Indian economy had expanded enough to Therefore , the correct answer is D) Nuclear power
withstand them. This implies that the economic development.
sanctions did not severely impact India's economy.
Therefore, it cannot be concluded from the passage PASSAGE III
that the nuclear tests caused economic sanctions that
severely impacted India's economy. 11. Correct Option A
(H & S)
A. Temporary improvement in cancer symptoms.
7. Correct Option B
(H & S) Explanation: The author mentions "long before
The correct answer is option B) . According to the remissions became tenable for many," implying that
passage, the civil nuclear agreement, promoted by
remissions refer to a temporary improvement in
George Bush Jr. and Manmohan Singh, made India a
de facto nuclear power. This agreement helped cancer symptoms.
alleviate the anxiety surrounding India's nuclear
weapon capabilities.
12. Correct Option A

8. Correct Option C (H & S)


(H & S) A. Cancer patients in the early 20th century had few
The correct answer is option C).The passage states treatment options. Explanation: The passage
that China's blocking of India's membership in the
Nuclear Suppliers Group undermined the idea that discusses the lack of treatment options for cancer
the US and the West were the only ones to have patients in the early 20th century, which supports
double standards when it came to non-proliferation. statement A.

9. Correct Option D
(H & S) 13. Correct Option A
The correct answer is option D). The passage (H & S)
mentions that there was genuine danger that
A. Neutral Explanation: The passage presents
extremist organizations may obtain access to
Pakistan's nuclear weapons. This concern highlights information about the history of cancer treatment and
the potential security risks associated with Pakistan's the use of war metaphors without expressing a strong
nuclear arsenal.
emotional bias one way or another.

10. Correct Option D


(H & S) 14. Correct Option A
The word "nonproliferation" refers to the act of (H & S)
preventing the spread or dissemination of nuclear
A. Metaphor Explanation: The phrase "lawless
weapons or related technologies. In the given
passage, India is described as being an advocate for growth" uses a metaphor to compare the behavior of
nonproliferation in the international world. To find cancer cells to a lawless person or group.
the best substitute for this term, we need to identify
an option that conveys a similar meaning.
Disarmament: Disarmament refers to the reduction 15. Correct Option D
or elimination of military weapons (H & S)
Nuclear testing: Nuclear testing involves conducting
Answer: D. Putting up a fight against cancer
experiments or tests related to nuclear weapons or
nuclear technology. Explanation: The phrase "putting up a fight against
Military expansion: Military expansion refers to the cancer" is an example of a hawkish word or phrase,
growth or increase in a country's military which is discouraged by physicians, psychologists,
capabilities.
and patient advocates.
[31]
20. Correct Option C
PASSAGE IV (H & S)
16. Correct Option B Answer: c) Neutral
(H & S) Explanation: The passage presents historical
b) It marked a shift in the Soviet approach to information without expressing a particular
religion. emotional tone or bias towards the events described.
Explanation: The passage mentions that the meeting
that Stalin called in September 1943 marked a PASSAGE V
significant shift in the Soviet approach to religion, 21. Correct Option A
which would have a lasting impact on religious life (H & S)
for Soviet people and their descendants in the Answer: a. Hardened
following decades. Explanation: The word "sclerotic" is used to describe
the Qajar dynasty ruling Iran. The word "sclerotic"
17. Correct Option B means hardened or rigid, indicating that the Qajar
(H & S) dynasty was resistant to change.
Answer: b) The Church was heavily persecuted by
the Soviet state. 22. Correct Option B
Explanation: The passage mentions that before the (H & S)
meeting in September 1943, the Russian Orthodox Answer: b. Indo-Iranian exchange fostered modern
Church had been decimated by the officially atheist nationalism
Soviet state, with many of its clergy imprisoned or Explanation: The passage discusses how cultural
dead. exchange between Iran and South Asia resulted in
the emergence of national literature, cultures, and
18. Correct Option C identities in Iran. It argues that this exchange
(H & S) fostered modern nationalism, rather than greater
Answer: c) It led to a temporary reconciliation cosmopolitanism.
between the Soviet state and the Church.
Explanation: The passage mentions that Stalin's 23. Correct Option B
proposal to revive Orthodoxy was a shift in the (H & S)
Soviet approach to religion, but it was seen as a Answer: b. Iranian intellectuals in the 19th century
temporary wartime measure transformed Persian into a national language
Explanation: The passage discusses how Iranian
19. Correct Option C intellectuals in the 19th century transformed Persian
(H & S) from the lingua franca of a centreless Persianate
Answer: c) To revive the Russian Orthodox Church. world into the language of national identity in the
Explanation: The passage mentions that Stalin called nation-state of Iran. It argues that Persian was
the meeting in September 1943 to propose the refashioned into a national language and literature,
revival of the Russian Orthodox Church, a move that positioning Iran as the proprietor of the Persianate
marked a significant shift in the Soviet approach to tradition.
religion.

[32]
with the discomfort associated with swinging rather
24. Correct Option A than universally accepting it as a therapeutic
(H & S) practice. Option D is incorrect as the passage does
Answer: a. Objective not suggest that the fear of falling prevented people
Explanation: The passage presents information about from enjoying swinging in many cultures throughout
the emergence of national literature, cultures, and history.
identities in Iran and South Asia, and the role of
cultural exchange in fostering modern nationalism. 2. Correct Option D
The tone is neutral and objective, without expressing (H & S)
a particular bias or opinion. Answer: D) Swinging machines were believed to
have therapeutic benefits by European and American
25. Correct Option C doctors until the end of the 18th century.
(H & S) Explanation: The passage mentions that European
Answer: c. Personification and American doctors believed that the sweating,
Explanation: The sentence personifies the Qajar retching or vomiting that accompanied swinging
dynasty ruling Iran, describing it as "sclerotic" and could be therapeutic until the end of the 18th
"incapable of resistance century. The passage also talks about the Czech
anatomist Jan Evangelista Purkyně who set up a
Section II rotating chair suspended by a rope, a device not
PASSAGE I unlike those used at the time for the treatment of
1. Correct Option C various forms of insanity, and suffered unspeakably
(H & S) while subjected to the rigours of swinging.
Answer: C) Swinging has undergone changes in its Therefore, it can be assumed that swinging machines
cultural significance over time and has been were believed to have therapeutic benefits by
associated with both positive and negative European and American doctors until the end of the
experiences. 18th century. Option A is incorrect as the passage
Explanation: The passage states that the uses of mentions that swinging was not always a positive
swinging have changed over time and have been experience and many who swung often experienced
associated with both positive and negative vertigo and dizziness. Option B is incorrect as the
experiences. While swinging was believed to have passage mentions that swinging machines were
therapeutic effects in the past, it also caused physical becoming popular in the parks and fairs of Bohemia.
discomfort such as vertigo, dizziness, sweating, Option C is incorrect as the passage mentions that
retching or vomiting. Additionally, swinging has Jan Evangelista Purkyně set up a rotating chair
been associated with horror in spiritualism and suspended by a rope, which was not unlike those
popular culture. Therefore, it can be concluded that used at the time for the treatment of various forms of
swinging has undergone changes in its cultural insanity, but it does not mention that he invented
significance over time and has been associated with swinging machines.
both positive and negative experiences. Option A is
incorrect as the passage states that swinging has not
always been a positive experience. Option B is
incorrect as the passage suggests that doctors worked

[33]
3. Correct Option A punishment for criminals, not the swing's original
(H & S) purpose. Option B is mentioned in the passage but
Answer: A) European and American doctors does not strengthen the argument that the swing was
believed that swinging could cure various forms of initially considered a device of death that became
insanity. apotropaic. Option D is mentioned in the passage but
Explanation: The passage mentions that well until is also irrelevant to the argument.
the end of the 18th century, European and American
doctors believed that the sweating, retching or 5. Correct Option C
vomiting that accompanied swinging could be (H & S)
therapeutic. This information supports option A as Answer: C) Swinging has evolved over time to
the correct answer. Option B is incorrect because the become an instrument of play and discipline, but it
passage clearly states that swinging was used as both has also been a source of physical discomfort and
an instrument of play and discipline and as a horror.
therapeutic practice. Option C is incorrect because Explanation: The passage discusses the history of
while the passage mentions that Jan Evangelista swinging, from its origins in punishment and therapy
Purkyně suffered greatly while swinging, it does not to its evolution as an instrument of play and
imply that he was the only doctor to experience discipline. It also highlights the negative physical
negative effects. Option D is incorrect because the effects of swinging and its association with horror in
passage mentions that the therapeutic effects of spiritualism and popular culture. Therefore, the main
swinging were believed in the past, indicating that argument presented in the passage is that while
the discomfort caused by swinging was not always swinging has evolved over time to become an
considered a negative outcome. instrument of play and discipline, it has also been a
source of physical discomfort and horror. Option C
4. Correct Option C accurately summarizes this argument. Options A, B,
(H & S) and D may touch upon specific aspects mentioned in
Answer: C) Athenians began swinging themselves the passage but do not capture the main argument
on wooden planks hung from ropes to ward off an presented in the passage as a whole.
evil spell, which prevented young Athenian girls
from hanging themselves. PASSAGE II
Explanation: The passage mentions that the swing 6. Correct Option B
was initially considered a device of death that (H & S)
became apotropaic, meaning it became capable of Answer: B) The fundamental attribution error can
warding off an evil spell. The passage goes on to lead to moral condemnation of individuals.
mention that Athenians ended a sad epidemic by Explanation: The passage explicitly states that the
instituting the practice of swinging themselves while fundamental attribution error can have both moral
seated on wooden planks hung from ropes, which and legal consequences. Specifically, when behavior
prevented young Athenian girls from hanging is attributed to a person's character or disposition, it
themselves. Therefore, statement C strengthens the can result in moral condemnation, whereas
argument that the swing was initially considered a attributing behavior to situational factors can
device of death that became apotropaic. Option A is exculpate or excuse a transgressor. Therefore, option
irrelevant to the argument, as it talks about a form of B is the correct answer.

[34]
Option A is incorrect because the passage does not situation. This argument is more likely to lead to a
suggest that people tend to attribute behavior to a harsher sentence for the convicted criminal.
person's character because they lack information Option C, where the judge and jury attribute the
about the situation. In fact, the passage suggests that crime to the defendant's disposition rather than the
people tend to neglect the importance of situations situational factors, is also an example of the
and focus more on personal dispositions when fundamental attribution error. This attribution is
explaining behavior. more likely to lead to a harsher sentence for the
Option C is incorrect because the passage does not convicted criminal.
suggest that the fundamental attribution error is more Option D, where the defendant emphasizes that they
likely to occur when people have a lot of information have undergone significant personal growth since
about situational constraints. In fact, the passage committing the crime, is an example of emphasizing
suggests that even in situations where there are the role of the person as the cause of the crime. This
prominent situational constraints, people still tend to argument is less likely to lead to a reduced sentence
attribute behavior to personal dispositions. for the convicted criminal, as it attributes the
Option D is incorrect because the passage clearly behavior to the person rather than the situation.
suggests that the fundamental attribution error is
both morally and legally consequential, implying 8. Correct Option A
that it is relevant in both legal and non-legal (H & S)
contexts. Answer: A. Criminals are more likely to receive
harsher sentences when situational factors are taken
7. Correct Option A into account.
(H & S) Explanation: The passage explains that the
Answer: A. The defense argues that the defendant fundamental attribution error can have significant
had no choice but to commit the crime due to legal consequences. When people tend to attribute
extreme situational constraints. wrongdoing more to the person, arguments about
Explanation: The passage explains that the complete personal change may be more persuasive.
fundamental attribution error can have significant In contrast, situational attributions can help
legal consequences. When people tend to attribute exculpate or excuse a transgressor. However, the
wrongdoing more to the person, arguments about passage also notes that the fundamental attribution
complete personal change may be more persuasive. error tends to inflate the importance of dispositions
In contrast, situational attributions can help and neglect the importance of situations. This means
exculpate or excuse a transgressor. Therefore, in a that judges and juries are more likely to consider
legal setting, if the defense can successfully argue personal dispositions rather than situational factors
that the defendant had no choice but to commit the when making sentencing decisions. Therefore,
crime due to extreme situational constraints, it is criminals are more likely to receive harsher
most likely to lead to a reduced sentence for the sentences when situational factors are taken into
convicted criminal. This option is the correct answer. account. This option is the correct answer.
Option B, where the prosecution argues that the Option B, where judges and juries are more likely to
defendant's character is fundamentally flawed and consider situational factors when making sentencing
that they are likely to reoffend, is an example of decisions, contradicts the implication of the
attributing the behavior to the person rather than the

[35]
fundamental attribution error explained in the factors can contribute to behavior, it does not claim
passage. Therefore, it is an incorrect answer. that they are the primary cause.
Option C, where the use of situational attributions in Option C, which suggests that personal dispositions
legal settings can lead to moral condemnation of the are the primary cause of behavior, is also not a
defendant, is not supported by the passage. The complete or accurate representation of the argument
passage explains that situational attributions can help presented in the passage. While the passage notes
exculpate or excuse a transgressor, leading to that personal dispositions can contribute to behavior,
reduced moral condemnation. it does not claim that they are the primary cause.
Option D, where defendants who emphasize personal Option D, which suggests that the use of situational
growth are more likely to receive reduced sentences, attributions in legal settings is the best way to
is a partial implication of the passage. While exculpate transgressors and promote moral
emphasizing personal growth may be persuasive in rehabilitation, is not supported by the passage. While
some cases, the passage also notes that arguments the passage notes that situational attributions can
about complete personal change tend to emphasize help exculpate or excuse a transgressor, it also notes
the role of the person as the cause of the crime, that the fundamental attribution error tends to inflate
which can lead to harsher sentences. Therefore, the importance of personal dispositions in legal
option A is a more accurate and complete settings. Therefore, option A is the correct answer.
implication of the passage.
10. Correct Option C
9. Correct Option A (H & S)
(H & S) Answer: C. Consider both situational factors and
Answer: A. The fundamental attribution error is a personal disposition attributions in moral and legal
well-known phenomenon in social psychology that judgments.
affects how people explain behavior and has Explanation: The passage discusses the tendency of
significant moral and legal consequences. individuals to attribute behavior to personal
Explanation: The passage's main argument is that the dispositions and neglect the importance of situational
fundamental attribution error, which refers to the factors, known as the fundamental attribution error.
tendency to attribute behavior to personal It also highlights the moral and legal implications of
dispositions while neglecting situational factors, has these attributions, including their potential for moral
significant moral and legal consequences. The condemnation and harsh legal consequences.
passage discusses the history and prevalence of this Option A, which suggests always prioritizing
phenomenon and presents evidence from situational factors over personal disposition
experiments to support the argument. It also attributions, is incorrect because it implies a bias in
highlights the difference in moral and legal the opposite direction, which may not reflect the
implications between attributing behavior to reality of the situation. Sometimes personal
personal dispositions versus situational factors. dispositions may play a significant role in a person's
Option B, which suggests that situational factors are behavior, and it would not be appropriate to
the primary cause of behavior, is not a complete or overlook this completely.
accurate representation of the argument presented in Option B, which suggests avoiding making any
the passage. While the passage notes that situational moral or legal judgments about individuals'
behavior, is also incorrect because moral and legal

[36]
judgments are necessary in some situations and are a as it misrepresents the Stoic philosophy by implying
part of our justice system. It is important to make that it does not accept the past and future, while
informed judgments that take into account all option C is incorrect as it only focuses on one aspect
relevant factors. of each philosophy. Option D is also incorrect as it
Option D, which suggests consciously overlooking repeats the claim made in the passage without
situational factors in moral and legal judgments, is highlighting the difference between the two
incorrect because this would perpetuate the philosophies.
fundamental attribution error and could lead to unfair
judgments. It is important to consider all factors, 12. Correct Option C
including situational factors, when making (H & S)
judgments about behavior. Answer: C) Shugendō philosophy and mindfulness
Option C, which suggests considering both practices have some similarities in terms of living in
situational factors and personal disposition the present, but Shugendō places more value on
attributions in moral and legal judgments, is the best activities that directly involve the present moment.
answer. By considering both factors, individuals can Explanation: The passage states that Shugendō
avoid the fundamental attribution error and make practices of living in the moment, such as walking
more informed and fair judgments. It is important to meditation and tokogatame meditation, may seem
take into account the circumstances surrounding a similar to current mindfulness practices inspired by
behavior and the personal tendencies of the Stoicism. However, the passage goes on to explain
individual when making moral and legal judgments. that the Stoic philosophy of living in the present is
incomplete, and that according to Shugendō
PASSAGE III philosophy, the idea of living in the moment as
11. Correct Option B expressed by Stoic philosophers is contradictory.
(H & S) The passage also describes how writing and reading
Answer: B. The Shugendō philosophy believes that are not considered activities that involve living in the
writing and reading can distract one from living in present in Shugendō philosophy. Therefore, it can be
the moment, while the Stoic philosophy sees these inferred that while there are similarities between
activities as helpful for reflection and acceptance. Shugendō philosophy and mindfulness practices in
Explanation: The passage states that while the Stoic terms of living in the present, Shugendō places more
philosophy emphasizes living in the present and value on activities that directly involve the present
accepting the uncertainty of the future, it is moment.
incomplete because it does not address the
distractions of writing and speaking. In contrast, the 13. Correct Option A
Shugendō philosophy sees writing and reading as (H & S)
taking one's mind away from the present moment, Answer: A) Writing is a distraction from living in
and emphasizes physical training and walking the present moment.
meditation as ways to truly live in the moment. Explanation: The passage states that according to the
Therefore, option B is the correct answer as it Shugendō philosophy, writing and reading can take
captures the main difference between the two our minds away from the world, away from
philosophies regarding the role of writing and concentrating on our training, and away from the
reading in living in the present. Option A is incorrect present moment. The Shugendō philosophy rejects

[37]
the Stoic model of living in the present because it If Shugendō's practices are more intense, it suggests
believes that the Stoic philosophy of living in the that they are more effective in helping practitioners
present is incomplete. However, the passage does to live in the present moment and solidify their place
not suggest that mindfulness practices are not in the world. This supports the argument that
effective for achieving acceptance or that living in Shugendō's philosophy of living in the present is
the present moment is the only way to achieve more complete than Stoicism's.
acceptance. Therefore, options B and D are
incorrect. Option C may seem relevant because the 15. Correct Option B
passage mentions that the Stoic philosophy of living (H & S)
in the present is incomplete, but it is not necessary Answer: B) Some Shugendō practitioners engage in
for the Shugendō philosophy to reject the Stoic self-reflection and journaling to reflect on their
model. Instead, the passage highlights the training and personal growth.
assumption that writing is a distraction from living in Explanation: The argument in the passage is that the
the present moment, which is necessary for the Stoic philosophy of living in the present is
Shugendō philosophy to reject the Stoic model of incomplete according to Shugendō philosophy
living in the present. because Stoic philosophers like Seneca and Marcus
Aurelius wrote down their thoughts, which suggests
14. Correct Option C they did not constantly live in the moment. The
(H & S) passage also states that in Shugendō, there is no
Answer: C) Shugendō training includes meditation place for self-reflection through journaling as it
practices that are more intense than those of distracts practitioners from living in the moment.
Stoicism. Option B weakens this argument by suggesting that
Explanation: The passage argues that while the Stoic some Shugendō practitioners do engage in self-
philosophy of living in the present is built around reflection and journaling, which contradicts the idea
acceptance and mindfulness, it is incomplete. The that there is no place for such practices in Shugendō.
Shugendō philosophy of living in the present is If self-reflection and journaling are considered
presented as more complete, but the passage does not acceptable in some form in Shugendō, then the
explicitly state why. To strengthen the argument, we argument that the Stoic philosophy is incomplete
need to find a statement that highlights a difference according to Shugendō because of the practice of
between Shugendō and Stoicism that supports the writing down thoughts is weakened.
claim of Shugendō's superiority. Option A is irrelevant to the argument as it only
Option A is irrelevant because the length of a states that both Stoic and Shugendō philosophies
philosophy's history does not necessarily determine believe in the importance of living in the present,
its completeness. Option B actually weakens the which does not address the argument's main point.
argument by suggesting a potential limitation of the Option C actually strengthens the argument as it
Shugendō philosophy. Option D is also irrelevant acknowledges that both philosophies involve
because it does not relate to the argument that discipline and training to embrace the present
Shugendō is more complete than Stoicism. moment, which supports the idea that the Stoic
Option C strengthens the argument because it philosophy is incomplete according to Shugendō.
highlights a difference in the intensity of the
meditation practices between the two philosophies.

[38]
Option D is also irrelevant to the argument as it does imagination itself turns into a form of reason.
not address the main point of the argument regarding Through this process, poetry creates an aesthetic idea
the practice of writing down thoughts in Stoicism. that prompts a wealth of thought to which no
concept can be adequate, and which no language can
PASSAGE IV name. According to Kant, aesthetic ideas are
16. Correct Option C concepts that reach for the transcendent while
(H & S) remaining grounded in experience, but give up on
Answer: C) For Kant, the imagination's role in being concepts. Therefore, it can be inferred that
poetry is to provide a concrete grounding for Kant believed that poetry is a means of bridging the
transcendent concepts. gap between reason and experience in order to gain
Explanation: According to the passage, Kant saw insight into transcendent concepts, making option C
poetry as a means for bringing transcendent concepts the correct answer.
down to a level where they could be more concretely Option A is incorrect because the passage states that
understood through the imagination. He believed Kant viewed aesthetic ideas as reaching for the
that aesthetic ideas created through poetry were the transcendent while remaining grounded in
counterparts of ideas of reason, and that they reached experience. Option B is incorrect because the
for the transcendent while remaining grounded in passage does not suggest that poetry is the only way
experience. Kant saw poetry as a way to expand the to fully comprehend transcendent concepts, but
implications and affinities of a concept, opening the rather a means of gaining insight into them. Option
mind to an "immeasurable field of interrelated D is incorrect because the passage describes how
thoughts." Therefore, option C is the correct answer. poetry can reach into the ineffable that binds the
Option A is incorrect because Kant believed that "mere words" of a concept, be it "God" or "envy" or
poetry was capable of conveying transcendent "dog," to its life in thought and feeling, indicating
concepts. Option B is incorrect because while Kant that poetry can contribute to understanding
saw the poetic faculty as an analogue to reason, he transcendent concepts.
did not view it as a substitute for reason. Option D is
incorrect because Kant did not believe that poetry 18. Correct Option B
unravels complex concepts into more tangible forms, (H & S)
but rather that it brings transcendent concepts down Answer: B) By discussing the relationship between
to a more tangible level while still remaining aesthetic ideas and concepts of reason
grounded in experience. Explanation: The passage provides support for the
argument that poetry plays a significant role in
17. Correct Option C understanding transcendent concepts according to
(H & S) Kant by discussing the relationship between
Answer: C) Poetry is a means of bridging the gap aesthetic ideas and concepts of reason. Kant believed
between reason and experience in order to gain that aesthetic ideas were the imagination's analogue
insight into transcendent concepts. to reason's world-making, unimaginable thoughts.
Explanation: The passage states that Kant viewed Aesthetic ideas, unlike concepts of reason, reach for
poetry as a way of taking a concept and dragging it the transcendent while remaining grounded in
to a place halfway between pure reason and experience. The passage explains that great poems
experience, where concepts fall apart but the take a concept and drag it to a place halfway

[39]
between pure reason and experience, where concepts Option B is the correct answer because it directly
fall apart but the imagination itself turns into a form contradicts the argument presented in the passage
of reason. Poetry provides a unique insight into that poetry plays a significant role in understanding
transcendent concepts that reason alone cannot transcendent concepts according to Kant. If reason is
achieve. Therefore, option B is the correct answer. the only reliable way to understand transcendent
Option A is incorrect because although the passage concepts, then the role of poetry in understanding
explains the limitations of reason in comprehending such concepts is minimized or eliminated.
transcendent concepts, it does not specifically Option C also weakens the argument presented in the
provide support for the argument that poetry plays a passage to some extent, but it goes beyond the scope
significant role in understanding transcendent of the argument. The passage assumes that
concepts. transcendent concepts can be explored, so this option
Option C is incorrect because although the passage seems less relevant to the argument.
mentions the exploration of transcendent concepts in Option D weakens the argument presented in the
poetry, it does not specifically provide support for passage. However, the question asks for the option
the argument that poetry plays a significant role in that weakens the argument the most. Option B
understanding transcendent concepts. weakens the argument more strongly than option D
Option D is incorrect because the passage does not because it directly contradicts the argument that
directly compare the effectiveness of poetry and poetry plays a significant role in understanding
reason in understanding transcendent concepts. transcendent concepts according to Kant. Option D
only suggests that aesthetic ideas may not provide
19. Correct Option B any new insight into transcendent concepts beyond
(H & S) what can be achieved through reason alone. This is a
Answer: B) Reason is the only reliable way to weaker form of weakening than what option B does,
understand transcendent concepts. which denies the significant role of poetry
Explanation: The argument presented in the passage altogether. Therefore, option B is a stronger answer
is that poetry plays a significant role in than option D.
understanding transcendent concepts according to
Kant. The passage explains that aesthetic ideas, 20. Correct Option C
unlike concepts of reason, reach for the transcendent (H & S)
while remaining grounded in experience, providing a Answer: C) The author contends that poetry plays a
unique insight into transcendent concepts that reason significant role in understanding transcendent
alone cannot achieve. Therefore, any option that concepts according to Kant.
weakens the idea that poetry plays a significant role Explanation: The passage discusses the view of Kant
in understanding transcendent concepts would be the on the role of poetry in understanding transcendent
correct answer. concepts. According to the passage, Kant believes
Option A does not weaken the argument presented in that aesthetic ideas, unlike concepts of reason, reach
the passage since it only talks about the fact that for the transcendent while remaining grounded in
many great works of literature do not explore experience. The passage further explains that
transcendent concepts. This does not imply that aesthetic ideas provide a unique insight into
poetry cannot play a significant role in transcendent concepts that reason alone cannot
understanding transcendent concepts. achieve.

[40]
The author's view on the role of poetry in major public buildings was a key component of
understanding transcendent concepts is presented in euergetism in the Greco-Roman world.
the passage as being in line with Kant's view. The Option A is incorrect because the passage states that
author contends that poetry plays a significant role in euergetism was a practice in the Greco-Roman
understanding transcendent concepts according to world, not just limited to the Roman Empire.
Kant. This is supported by the passage's explanation Option B is incorrect because the passage states that
that aesthetic ideas, as presented in poetry, provide a euergetism involved the transfer of wealth from the
unique insight into transcendent concepts that reason private to the public sphere, not the other way
alone cannot achieve. around.
Option A is incorrect because the author does not Option C is incorrect because the passage states that
suggest that poetry is the only reliable way to there were higher-order benefactions beyond public
understand transcendent concepts. The passage banquets and entertainments, such as the
acknowledges that reason is also important in beautification of public buildings, restoration of
understanding transcendent concepts. infrastructure, and establishment of perpetual
Option B is also incorrect because the author does endowments.
not argue for an equal role for poetry and reason in
understanding transcendent concepts. Instead, the 22. Correct Option B
author emphasizes the unique insights that poetry (H & S)
can provide. Answer: B) The forms of public giving in the Greco-
Option D is incorrect because the author clearly Roman world, such as financing public buildings and
disagrees with the idea that poetry has no role in banquets, were similar in nature and purpose to
understanding transcendent concepts. The author modern-day charitable donations and establishment
contends that poetry plays a significant role in of foundations.
understanding transcendent concepts according to Explanation: The correct answer can be inferred
Kant. from the second paragraph of the passage, which
states that there are indeed some similarities between
PASSAGE V ancient magnates and their modern counterparts
21. Correct Option D when it comes to public displays of generosity, such
(H & S) as high-profile charitable donations and
Answer: D) Major public buildings, such as temples establishment of major foundations. This suggests
and amphitheatres, were financed by local elites as that the forms of public giving in the Greco-Roman
part of euergetism. world were similar in nature and purpose to modern-
Explanation: The correct answer can be found in the day philanthropy.
fourth paragraph of the passage, where it is stated Option A is incorrect because the passage does not
that "The truly big-ticket items were the major public make any explicit comparison between the motives
buildings financed by local elites, the grand of euergetism in the Greco-Roman world and
monuments that travellers to any part of the Roman modern-day philanthropy. While the passage does
Empire still encounter today: temples, theatres and mention that euergetism involved the transfer of
amphitheatres, libraries, public baths, colonnades wealth from the private to the public sphere, it does
and basilicas." This shows that the financing of not make any definitive claims about the underlying
motives of this practice.

[41]
was a historically distinctive modality of public
Option C is incorrect because the passage does not giving.
suggest that euergetism in the Greco-Roman world Option B is incorrect because it does not address the
was a spontaneous and voluntary practice, or that question of whether euergetism was a historically
modern-day philanthropy is largely motivated by tax distinctive practice, but rather focuses on the social
incentives and public relations benefits. and political dynamics of the practice.
Option D is incorrect because the passage does not Option C is also incorrect because it does not
make any definitive claims about the religious or directly address the question of whether euergetism
secular motivations behind euergetism or modern- was a historically distinctive practice, but rather
day philanthropy. While the passage does mention describes the specific functions served by the public
that euergetism involved the financing of public buildings financed by local elites.
buildings and other structures that could please the Option D is the correct answer because it strengthens
gods, it does not suggest that this was the primary the argument by suggesting that euergetism played
motivation behind the practice. Similarly, while an important role in promoting social cohesion and
modern-day philanthropy can be driven by concerns civic identity in the Roman Empire. This suggests
such as social justice and environmental protection, that euergetism was not simply a practice of elite
the passage does not suggest that these are the only patronage or personal ambition, but rather a practice
or primary motivations behind modern-day giving. that contributed to the broader social and cultural
fabric of the Roman Empire.
23. Correct Option D
(H & S) 24. Correct Option B
Answer: D) The fact that euergetism was a major (H & S)
factor in promoting social cohesion and civic Answer: B) The wealthy elite engaged in euergetism
identity in the Roman Empire. primarily to promote their own political and social
Explanation: The argument presented in the passage interests.
is that euergetism in the Greco-Roman world was a Explanation: The passage suggests that euergetism
historically distinctive modality of public giving, was largely practiced by the wealthy elite in the
characterized by the transfer of wealth from the Greco-Roman world, who financed public buildings,
private to the public sphere through the financing of banquets, and other forms of civic benefaction as a
public buildings, banquets, and other forms of civic means of transferring their wealth from the private to
benefaction. To strengthen this argument, we need to the public sphere. The passage also notes that these
find an answer choice that supports the idea that practices were used to gain political power and
euergetism was not simply a variation on a broader influence. While the passage does not explicitly state
phenomenon of public giving that was common to that the motivations of the wealthy elite who
other ancient civilizations, but rather a unique and engaged in euergetism were to promote their own
historically specific practice that contributed to the political and social interests, this can be inferred
development of the Roman Empire. from the evidence presented in the passage.
Option A is incorrect because it weakens the Therefore, option B is the correct answer. Option A
argument by suggesting that similar forms of public is incorrect because the passage does not suggest that
giving were prevalent in other ancient civilizations, the wealthy elite engaged in euergetism primarily to
which would undermine the claim that euergetism gain personal fame and recognition. Option C is

[42]
incorrect because while the passage notes that Section III
euergetism involved a sense of civic duty and public PASSAGE I
spirit, this is not presented as the primary motivation 1. Correct Option A
of the wealthy elite who engaged in euergetism. (H & S)
Option D is incorrect because the passage provides A. 1974
information that allows for an inference to be made Explanation: India conducted its first nuclear bomb
about the motivations of the wealthy elite who test in Pokhran on May 18, 1974. The test was code-
engaged in euergetism. named "Smiling Buddha" and it was a peaceful
nuclear explosion.
25. Correct Option A
(H & S) 2. Correct Option A
Answer: A) Similar practices of public giving can be (H & S)
found in other ancient civilizations outside of the Answer: A. Smiling Buddha
Greco-Roman world. Explanation: The first nuclear bomb test conducted
Explanation: The passage argues that euergetism in by India in Pokhran in 1974 was code-named
the Greco-Roman world was a historically "Smiling Buddha". It was a successful test of a
distinctive form of public giving, involving the nuclear device and marked India's entry into the
transfer of wealth from the private to the public exclusive club of nuclear powers.
sphere, and motivated by a desire for political power
and influence. Option A, if true, would weaken this 3. Correct Option B
argument by suggesting that similar practices of (H & S)
public giving can be found in other ancient Answer: B. Three
civilizations outside of the Greco-Roman world. Explanation: India conducted a series of three
This would suggest that euergetism was not as nuclear bomb tests in Pokhran in May 1998.
unique as the passage suggests. Option B would not
weaken the argument, as the passage does not 4. Correct Option B
present personal fame and recognition as the primary (H & S)
motivation of the wealthy elite who engaged in Answer: B. Shakti-II
euergetism. Option C would not weaken the Shakti-II was the code name of the thermonuclear
argument, as the passage suggests that euergetism device tested by India in Pokhran in May 1998.
did involve the transfer of wealth from the private to
the public sphere. Option D would not weaken the 5. Correct Option B
argument, as the passage argues that the types of (H & S)
public buildings financed by the wealthy elite in the Answer: B. R Chidambaram
Greco-Roman world were unique in their scale and Explanation: R Chidambaram was the Chief
scope, not just in their types. Therefore, option A is Scientific Adviser to the Indian Prime Minister
the correct answer. during the Pokhran-II tests in May 1998. He played a
key role in designing and overseeing the tests.

[43]
PASSAGE II 1881 at Dakshineshwar's Kali Temple. The Indian
6. Correct Option A philosophies of Vedanta and Yoga were introduced
(H & S) to the Western World by him.
Ans- a) The Pre-monastic name of Swami
Vivekananda was Narendra Nath Dutta. He became PASSAGE III
a disciple of Saint Ramakrishna Paramhamsa after 11. Correct Option B
being introduced to him by one of his English (H & S)
professors in 1881 at Dakshineshwar's Kali Temple. Justice B V Nagarathna is likely to become the first
The Indian philosophies of Vedanta and Yoga were female Chief Justice of India in September 2027, as
introduced to the Western World by him. per the seniority list. However, her term as CJI will
be limited to only 36 days if the appointments follow
7. Correct Option C the seniority order.
(H & S) Hence, Option (B) is correct.
Ans- c) The National Youth Day 2023 theme is
"Viksit Yuva-Viksit Bharat. he Centre said the 12. Correct Option D
festival is held to provide exposure to youths at a (H & S)
national level and this year's theme for the four-day According to Article 124 - The Supreme Court of
programme is “Viksit Yuva – Viksit Bharat India shall consist of a Chief Justice of India and a
(Developed Youth – Developed India). maximum of seven other Judges, until the Parliament
decides to increase the number. The President of
8. Correct Option B India shall appoint every Judge of the Supreme
(H & S) Court by a warrant under his hand and seal, after
Ans-b) International Youth Day is commemorated consulting with the Judges of the Supreme Court and
every year on 12 August, bringing youth issues to High Courts in the States, as deemed necessary. The
the attention of the international community and Judges shall hold office until the age of sixty-five
celebrating the potential of youth as partners in years, and in the case of appointment of a Judge
today's global society. other than the Chief Justice, the Chief Justice of
India shall always be consulted. A Judge may resign
9. Correct Option D by addressing the President in writing, and a Judge
(H & S) may be removed from his office by the process of
Ans-d) Vivekananda is best known for his speech at impeachment.
the Parliament of the World's Religions in Chicago Hence, Option (D) is correct.
in 1893, when he began his speech by saying "Sisters
and brothers of America..." and introduced the 13. Correct Option A
culture of India, its importance, Hinduism, etc. (H & S)
In 1993, V. Ramaswami J became the first judge to
10. Correct Option C face impeachment proceedings, which were initiated
(H & S) in the Lok Sabha. However, the motion did not
Ans-c) Swami vivekanandha became a disciple of receive the necessary two-thirds majority and thus,
Saint Ramakrishna Paramhamsa after being he was not removed from his position as a judge.
introduced to him by one of his English professors in Hence, Option (A) is correct.

[44]
14. Correct Option C Hence, Option (D) is correct.
(H & S)
On March 10 2022 the International Day of Women PASSAGE VI
Judges was celebrated for the first time after a 16. Correct Option A
resolution was passed by the UN General Assembly (H & S)
in April 2021. The purpose of this day is to Option A
acknowledge the hard work and significant The nation's first PM Mega Integrated Textile
contributions made by women judges. It also serves Regions and Apparel (PM MITRA) Park is set to be
as a means to inspire and empower young women established in Virudhunagar District of Tamil Nadu,
and girls who have aspirations to become judges and and employ an estimated 2 lakh people, the
leaders in their communities. Memorandum of Understanding for which was
Hence, Option (C) is correct. signed between the Central and State governments.

15. Correct Option D 17. Correct Option A


(H & S) (H & S)
(D) ; Correct sequence is - PPP is an arrangement between government and
A-4 private sector for the provision of public assets
B-3 and/or public services. Public-private partnerships
C-2 allow large-scale government projects, such as roads,
D-1 bridges, or hospitals, to be completed with private
Article 217 - deals with appointments and conditions funding.
of appointment of High court judges 1. In this type of partnership, investments are
Article 125 - pertains to the payment of salaries and undertaken by the private sector entity, for a
allowances to Judges. According to this article, the specified period of time.
salaries of Supreme Court Judges shall be 2. These partnerships work well when private sector
determined by Parliament and until then, the salaries technology and innovation combine with public
specified in the Second Schedule shall be paid. sector incentives to complete work on time and
Article 127 - This article pertains to the appointment within budget.
of ad-hoc judges and states that they are appointed 3. As PPP involves full retention of responsibility
by the Chief Justice of India in consultation with the by the government for providing the services, it
President of India. Retired judges of the Supreme doesn‟t amount to privatization.
Court can also be appointed as ad-hoc judges but 4. There is a well defined allocation of risk between
they do not hold the status of regular judges. the private sector and the public entity.
Article 129 - confers the status of a court of record to 5. Private entity is chosen on the basis of open
the Supreme Court, which means that it has the competitive bidding and receives performance
power to maintain its own records and the decisions linked payments.
made by it are considered as legal precedents. 6. PPP route can be alternative in developing
Furthermore, the Supreme Court is vested with the countries where governments face various
power to punish for contempt of itself, in addition to constraints on borrowing money for important
all the other powers that a court of record usually projects.
possesses.

[45]
7. It can also give required expertise in planning or
executing large projects. PASSAGE V
21. Correct Option D
18. Correct Option A (H & S)
(H & S) Answer: D) 14
Option A Explanation: The Forum for India-Pacific Islands
The Centre has selected sites in Tamil Nadu, Cooperation (FIPIC) consists of 14 Pacific Island
Telangana, Karnataka, Maharashtra, Gujarat, Countries (PICs) as members.
Madhya Pradesh and Uttar Pradesh to set up new
textile parks under the PM Mega Integrated Textile 22. Correct Option B
Regions and Apparel (PM MITRA) scheme. The (H & S)
parks will be set up by 2026-27. The total outlay for Answer: b) To enhance India's relations with PICs in
the project is Rs 4,445 crore, though the initial various fields
allocation in the 2023-24 Budget is only Rs 200 Explanation: The objective of the FIPIC is to
crore. enhance India's relations with the Pacific Island
Countries (PICs) in various areas such as trade,
19. Correct Option A investment, tourism, education, health, agriculture,
(H & S) renewable energy, disaster management, and climate
Solutions change.
Option A
Textile sector is the 2nd largest provider of 23. Correct Option A
employment in India, after agriculture. It provides (H & S)
employment to an estimated 45 million people Answer: a) Fiji
directly and to another 60 million indirectly through Explanation: The first FIPIC summit was hosted by
allied activities. India is the 6th largest exporter of Fiji in 2014
textile and apparel in the world, with 4% share of the
global trade in textiles and apparel. India‟s textile 24. Correct Option C
and apparel exports (including handicrafts) stood at (H & S)
USD 44.4 billion in FY22, a 41% increase YoY. Answer: c) USD 570 million
India‟s textiles industry has around 4.5 crore Explanation: Based on 2021-22 data, the total annual
employed workers including 35.22 lakh handloom trade between India and Pacific Island countries is
workers across the country. valued at USD 570 million.

20. Correct Option B 25. Correct Option A


(H & S) (H & S)
Option B Answer: a) Papua New Guinea
Shri Piyush Goyal is the Current Minister for Textile Explanation: Papua New Guinea is India's biggest
in India. trade partner among the Pacific Island countries in
terms of value.

[46]
PASSAGE VIII modifications with counterparties, interbank entities,
26. Correct Option A and borrowers.
(H & S)
Answer: A. A benchmark interest rate Section IV
Explanation: LIBOR is a widely used global PASSAGE I
benchmark interest rate that represents the average 1. Correct Option D
interest rate at which banks estimate they can borrow (H & S)
from each other in the London interbank market for The RTI Act mandates that public authorities
specific time periods. provide access to information held by them, unless it
falls within the specified exemptions. The economic
27. Correct Option C interests of the country are not explicitly mentioned
(H & S) as an exemption, and therefore, the PIO's decision to
Answer: C. SOFR deny the request based on economic interests is not
Explanation: In 2017, the U.S. Federal Reserve justified. Therefore the correct answer is d a) and b)
introduced the Secured Overnight Financing Rate are incorrect because the economic interests of the
(SOFR) as an alternative to LIBOR. country do not automatically override the right to
access information. The PIO must disclose
28. Correct Option D information unless it falls within the specified
(H & S) exemptions.
Solutions c) is incorrect because it falsely implies that the
Answer: D. All of the above citizen's right to access information can be denied
Explanation: The RBI is moving away from LIBOR based on economic interests, which is not supported
due to concerns over its reliability and integrity, its by the RTI Act.Hence the correct answer is d
heavy reliance on banks, and its susceptibility to
market manipulation. 2. Correct Option A
(H & S)
29. Correct Option B The correct answer is a) Yes, because the disclosure
(H & S) of information about ongoing investigations can
Answer: B. Adjusted SOFR rates hinder the investigative process.
Explanation: MMIFOR incorporates the adjusted The RTI Act includes exemptions to the disclosure
SOFR rates, which are compounded retrospectively of information, and one of the exemptions is for
for different time periods and obtained from the ongoing investigations. Disclosing information about
Bloomberg Index Services, among other ongoing investigations can potentially interfere with
components. the investigative process, and therefore, the denial of
the request based on this ground is justified.
30. Correct Option D b) is incorrect because it suggests that protecting the
(H & S) integrity of ongoing investigations takes precedence
Answer: D. All of the above over the right to information, which is not explicitly
Explanation: Transitioning from LIBOR to an ARR stated in the RTI Act.
poses challenges in technology and legal aspects, c) is incorrect because it disregards the exemption
dealing with existing contracts, making necessary for ongoing investigations mentioned in the RTI Act.

[47]
d) is incorrect because the RTI Act provides 5. Correct Option C
exemptions for ongoing investigations. (H & S)
The correct answer is c) No, because the RTI Act
3. Correct Option B mandates public authorities to disseminate
(H & S) information in an easily accessible manner.
The correct answer is b) Yes, because the citizen has Section 4(3) of the RTI Act emphasizes the wide
the right to hold the public authority accountable for dissemination of information in an easily accessible
non-compliance with the RTI Act. manner. While cost-effectiveness and effectiveness
Section 4(1) of the RTI Act mandates that public are important considerations, the public authority
authorities publish certain particulars within 120 cannot refuse to provide information solely based on
days of the Act's enactment. If a public authority the lack of resources for dissemination.a) and b) are
fails to comply with this obligation, a citizen can file incorrect because they wrongly suggest that the lack
a complaint to ensure accountability. of resources can justify the refusal to disseminate
a) is incorrect because it only mentions the legal information, which is not supported by the RTI Act.
obligation of the public authority without d) is incorrect because it assumes that the public
considering the citizen's right to hold them authority's lack of resources should
accountable.
c) and d) are incorrect because they downplay the PASSAGE II
significance of holding public authorities 6. Correct Option B
accountable for non-compliance with the RTI Act. (H & S)
Answer: b) This law is valid as sacrificing children is
4. Correct Option A an immoral act and Article 25 is subject to morality.
(H & S) Reason: The law prohibiting the practice of throwing
The correct answer is a) Yes, because the citizen has children into an open fire as a religious ceremony is
the right to appeal if the public authority fails to valid because it aims to protect the fundamental
respond within the specified timeframe. rights and well-being of children. Article 25
Section 5(2) of the RTI Act provides for the guarantees the freedom of religion but is subject to
designation of Assistant PIOs who receive public order, health, and morality. In this case, the
applications and appeals. If a citizen does not receive act of sacrificing children is considered an immoral
a response from the PIO within the specified act and goes against the principles of public morality
timeframe, they have the right to file an appeal. and the rights of the child, outweighing the right to
b) is incorrect because it incorrectly suggests that the freedom of religion
citizen can seek remedies for non-compliance with Hence, the correct option is (B)
the Act, but the primary remedy in this case is the
right to appeal. 7. Correct Option B
c) is incorrect because it falsely states that the RTI (H & S)
Act does not provide for the right to appeal in case of Answer: b) This law is valid as consuming drugs is a
non-response. health concern and Article 25 is subject to public
d) is incorrect because it wrongly assumes that the health.
specified timeframe is not legally binding. Reason: The law prohibiting the consumption of
drugs as part of a religious ceremony is valid

[48]
because it addresses health concerns. Article 25 blood is considered immoral, and restrictions can be
guarantees the freedom of religion but is subject to imposed on such practices in the interest of public
public order, health, and morality. In this case, the morality
consumption of drugs poses health risks, and the Hence, the correct option is (B)
state has a legitimate interest in protecting public
health and preventing potential harm associated with 10. Correct Option A
drug use, which outweighs the right to freedom of (H & S)
religion Answer: a) There is a violation of Article 26
Hence, the correct option is (B) (Freedom to manage religious affairs), which
includes the purchase of immovable property.
8. Correct Option D Reason: There is a violation of Article 26 in this
(H & S) case. Article 26 guarantees the freedom to manage
Answer: d) religious rights under Article 28 are religious affairs, including the right to form and
violated here as it permits educational institutions maintain institutions for religious and charitable
that are maintained by religious groups to purposes. The Pontie community's right to purchase
disseminate religious instruction. immovable property is an aspect of managing their
Reason: The right violated in this case is the religious affairs. By preventing them from
religious rights under Article 28. Article 28 purchasing the property, their right under Article 26
guarantees freedom as to attendance at religious is violated
instruction or religious worship in certain Hence, the correct option is (A)
educational institutions. It permits educational
institutions maintained by religious groups to PASSAGE III
disseminate religious instruction. By prohibiting the 11. Correct Option D
PCN community from disseminating religious (H & S)
knowledge in educational institutions managed by d) No, because the Act does not provide any remedy
the community, their right to religious instruction, as in case no PIOs are appointed.
protected under Article 28, is violated According to the scenario and the information in the
Hence, the correct option is (D) passage, if no Public Information Officers (PIOs)
have been appointed at the central level, there is no
9. Correct Option B specific provision in the Right to Information Act,
(H & S) 2005 that addresses this issue. Therefore, Mr.
Answer: b) This law is valid as religious freedoms Sharma would not be able to seek relief under the
under Article 25 are subject to morality, and this Act in such a situation.Option a) is incorrect because
practice is immoral. while the Act guarantees the right to information at
Reason: The law prohibiting the drinking of human both the central and state levels, it does not explicitly
blood as a religious practice is valid as religious address the scenario of no PIOs being appointed.
freedoms under Article 25 are subject to morality. Option b) is incorrect because although citizens can
While Article 25 guarantees the freedom of approach the Central Information Commission, it
conscience, profession, practice, and propagation of does not specifically address the scenario of no PIOs
religion, it is also subject to public order, health, and being appointed.
morality. In this case, the act of drinking human

[49]
Option c) is incorrect because Mr. Sharma's inability Option c) is incorrect because Mr. Patel's
to find a PIO does not imply that he has not responsibility is to wait for the response within the
exhausted all possible avenues. The scenario given time frame, and following up is not a
indicates that no PIOs have been appointed, making requirement in this scenario.
it impossible for Mr. Sharma to submit his request. Option d) is incorrect because the Act does impose
an obligation on the officers to respond within a
12. Correct Option A specific time frame, making this answer incorrect.
(H & S)
Answer: a) Yes, because the Act grants citizens the 14. Correct Option A
right to access information upon request. (H & S)
Explanation: Based on the scenario and the passage Solutions
information, Ms. Singh has the right to access Answer: a) Yes, because the Act prohibits
information upon request under the Right to unreasonable fee requirements for accessing
Information Act, 2005. If she was denied access by information.
the appointed officers, she can seek redress under the Explanation: According to the scenario and the
Act. Option b) is incorrect because while the Act passage information, if Mr. Khan was required to
empowers the Chief Information Commissioner to pay an unreasonable amount of fee for accessing the
enforce access to information, it does not specifically requested information, he can challenge the fee
address the scenario of denial by appointed officers. charged under the Right to Information Act, 2005.
Option c) is incorrect because the scenario does not The Act prohibits unreasonable fee requirements for
mention whether Ms. Singh provided sufficient accessing information.Option b) is incorrect because
grounds for requesting the information or not. while the Act empowers the Chief Information
Option d) is incorrect because the Act does provide Commissioner to review and modify fees, it does not
recourse for denial of access to information, making specifically address the scenario of challenging
this answer incorrect. unreasonable fees.
Option c) is incorrect because negotiating the fee
13. Correct Option B with the appointed officers is not a requirement or
(H & S) solution mentioned in the scenario.
Answer: b) Yes, because the Act allows citizens to Option d) is incorrect because the Act does not allow
file a complaint with the Central Information officers to determine the fee based on their
Commission. discretion, making this answer incorrect.
Explanation: According to the scenario and the
passage information, if Mr. Patel did not receive any 15. Correct Option C
response within the specified time frame mentioned (H & S)
in the legislation, he can take action under the Right Answer: c) No, because Mr. Gupta should have
to Information Act, 2005 by filing a complaint with provided evidence to prove the falsity of the
the Central Information Commission.Option a) is information.
incorrect because while the Act guarantees a timely Explanation: Based on the scenario and the passage
response to information requests, it does not information, if Mr. Gupta believes that the
explicitly address the scenario of no response within information provided by the officers is false, he
the specified time frame. would need to provide evidence to prove its falsity.

[50]
The Act does not explicitly hold officers accountable Explanation: If the arbitral tribunal refuses to grant
for the accuracy of the information they provide interim measures, Party A can approach the court for
unless there is evidence to prove it false.Option a) is interim relief, but it would need to demonstrate that
incorrect because while the Act guarantees the the arbitral tribunal's decision was biased. Option A
accuracy and authenticity of the provided is incorrect because the court's jurisdiction is not
information, it does not explicitly address the exclusive over interim relief matters, as parties can
scenario of false information. also seek such relief from the arbitral tribunal.
Option b) is incorrect because while the Act Option C is incorrect because the court's jurisdiction
empowers the Chief Information Commissioner to is not limited to enforcing arbitral awards, and
investigate false information, it does not specifically interim relief falls within its purview. Option D is
address the scenario of false information provided by incorrect as the technical nature of the subject matter
officers. does not determine the party's right to seek interim
Option d) is incorrect because the Act does not relief from the court.
specifically hold officers accountable for the
accuracy of the information they provide, making 18. Correct Option B
this answer incorrect. (H & S)
Explanation: If the parties fail to agree on the
PASSAGE IV number of arbitrators involved, they will have to
16. Correct Option B reach a mutual agreement on the number before
(H & S) arbitration can proceed. Option A is incorrect
The correct answer is b. If the governing law because the Chief Justice of the Supreme Court does
regarding arbitration proceedings is not clearly not automatically appoint a single arbitrator in such
defined in the arbitration clause, the parties will have cases. Option C is incorrect as the arbitration clause
to reach a mutual agreement on the governing law would not be deemed invalid solely based on the
before arbitration can proceed. This is because the lack of agreement on the number of arbitrators.
governing law sets the rules and procedures for the Option D is incorrect as there are no default numbers
arbitration process. Option A is incorrect because the of arbitrators specified in the question or the passage.
arbitration clause would not automatically be Hence, option B is correct
deemed invalid, but the lack of clarity on the
governing law may affect the arbitration process. 19. Correct Option C
Option C is incorrect as the Chief Justice's (H & S)
involvement is not required in this scenario. Option Answer: C) No, because an arbitral award is
D is incorrect as the court does not determine the regarded as final and binding, and the grounds for
governing law; it is decided by the parties or as per setting it aside are limited.
the arbitration law. Explanation: An arbitral award is generally regarded
as final and binding on both parties. The grounds for
17. Correct Option B setting aside an arbitral award under Section 34 are
(H & S) limited, such as when the arbitrator is biased, the
Answer: B) Yes, if Party A can demonstrate that the award is against public policy, or the arbitrator has
arbitral tribunal's decision is biased. exceeded their jurisdiction. Option A is incorrect
because bias alone may not be sufficient grounds to

[51]
set aside an arbitral award unless it is proven. Option to the submission of keys to the wrecking ball
B is incorrect as the violation of the governing law machine. Therefore the correct answer is D
alone may not be sufficient grounds to set aside the
award unless it falls within the limited grounds 22. Correct Option D
specified in Section 34. Option D is incorrect as the (H & S)
Chief Justice's involvement is not necessary for In order for the order to be valid, there must be
setting aside arbitral awards under Section 34. concrete evidence establishing the presence of
misinformation and its potential harm. Without
20. Correct Option B adequate evidence, the Executive Magistrate lacks
(H & S) the authority to issue an order that restricts the
Answer: B) Party A can request the arbitral tribunal distribution of a newspaper. Therefore the correct
to select an arbitrator from a list of arbitrators. answer is D.
Explanation: If Party A fails to comply with the
agreed-upon process of elimination for arbitrator 23. Correct Option D
selection, Party A can request the arbitral tribunal to (H & S)
select an arbitrator from a list of arbitrators. Option In a democratic society, the freedom of speech and
A is incorrect as approaching the Chief Justice for expression is a fundamental right that is protected
the appointment of an arbitrator would be applicable unless there are specific legal provisions or
only in case of a failure of the agreed mechanism for justifiable grounds for its restriction. Generally, the
appointment. Option C is incorrect as initiating authority to impose restrictions on freedom of
litigation proceedings would not be the appropriate speech and expression is vested in higher-level
action in this scenario. Option D is incorrect as Party authorities, such as the District Magistrate or higher
A cannot unilaterally choose an arbitrator without judicial bodies, rather than sub-divisional
following the agreed-upon process. Magistrates. In this case, the sub-divisional
Magistrate exceeds their authority by issuing an
PASSAGE V order that restricts the freedom of speech and
21. Correct Option D expression.
(H & S) Therefore option d is the correct answer
Section 144 grants certain magistrates the power to
issue orders in urgent cases to prevent disturbances 24. Correct Option B
or dangers to public peace, safety, and tranquility. (H & S)
However, the grounds for issuing such orders are In this scenario, the District Magistrate (DM) of
limited to specific situations, such as obstruction, Dholakpur issued an order restricting access to a 5
annoyance, injury to a person lawfully employed, km radius around the King's palace in response to
danger to human life, health, safety, disturbance of the information received about a potential bomb
public tranquility, or a riot. The passage does not threat. However, the order lacks sufficient
provide any indication that the CEO's threat of justification or the full reason for its issuance. When
destroying Company A's buildings falls within the issuing such restrictive orders, it is essential for the
grounds specified in Section 144. Therefore, it can DM to provide clear and specific justifications for
be argued that the sub-divisional magistrate the action taken. The order should include a
exceeded their authority by issuing an order related comprehensive explanation of the potential threat,

[52]
the necessity of the restriction, and the evidence or Explanation: A contract of indemnity must have a
intelligence supporting the decision. Without lawful object and purpose. Engaging in illegal
sufficient justification or a complete explanation, the activities goes against public policy and cannot be a
order becomes questionable in terms of its validity. valid purpose for a contract of indemnity. Option a is
Therefore option B is the correct answer incorrect because the validity of a contract of
indemnity is not solely based on mutual consent.
25. Correct Option D Option b is incorrect as the necessity of illegal
(H & S) activities does not make the contract enforceable.
In this scenario, the Commissioner of a city issues an Option d is incorrect as engaging in illegal activities
order imposing a curfew from 10:00 PM to 5:00 AM within legal boundaries is not possible.
in response to a rise in criminal activities during
nighttime. However, the authority to issue orders 28. Correct Option C
imposing a curfew lies with District or executive (H & S)
magistrate and since it is not explicitly mentioned Option c is correct because an indemnity amount
that the Commissioner was allowed by state must be reasonable and within the scope of the
government to do so option d is the correct answer contract's terms. In a contract of indemnity, the
Hence the correct option is D. amount of indemnification should be determined
based on the potential risks and losses that may arise
PASSAGE VI from the specified events or circumstances. Without
26. Correct Option C a specified limit, the courts would typically expect
(H & S) the indemnity amount to be reasonable and
Answer: c) No, because a contract of indemnity must proportionate to the potential losses that may occur.
include a lawful consideration for it to be valid. Therefore, R cannot claim unlimited indemnification
Explanation: A contract of indemnity, like any other from S in the absence of a specified limit, as the
contract, requires a lawful consideration for it to be indemnity amount must adhere to the principles of
valid. Consideration is the exchange of something of reasonableness and within the scope of the contract's
value between the parties. Without consideration, the terms.Option d is incorrect because the absence of a
contract of indemnity lacks a necessary element for specified limit does not automatically allow for
enforceability. Option a is incorrect as consideration unlimited indemnification. Indemnification amounts
is essential for the validity of the contract. Option b need to be reasonable and proportionate to the
is incorrect as intention alone is not sufficient for a potential losses that may occur. Option b is incorrect
contract to be valid. Option d is incorrect as mutual because the burden is not on R to demonstrate that
agreement on the absence of consideration does not the loss exceeds a reasonable limit. Option c is
fulfill the requirement for a valid contract. incorrect because unlimited indemnification would
need to be explicitly agreed upon in the contract for
27. Correct Option C R to claim such entitlement.
(H & S)
Answer: c) No, because a contract of indemnity
cannot be construed as a contract to engage in
unlawful behavior.

[53]
29. Correct Option A Wednesday 750
(H & S) Thursday 300
Answer: a) No, because a contract of indemnity does Friday 900
not cover losses caused by the negligence of the Total non defective phones sold on Wednesday
indemnity holder. = (1- 0.2)750 = 600
Explanation: Option c is incorrect because the
indemnifier is not responsible for losses caused by 2. Correct Option B
the negligence of the indemnity holder. Option d is (H & S)
incorrect as explicit agreement to cover losses Mobile phone sold on Monday
resulting from the indemnity holder's negligence is  60 
   2700  450
not a common requirement. Option c is incorrect  360 
because coverage for losses resulting from the Similarly, we can find the values of all the days.
indemnity holder's negligence would need to be Days Phones
explicitly included in the contract sold
Monday 450
30. Correct Option C Tuesday 300
(H & S) Wednesday 750
c) No, because an implied contract of indemnity only Thursday 300
applies to losses caused directly by the indemnifier.
Friday 900
Explanation: An implied contract of indemnity arises
from the conduct of the parties, but it still applies to
Required ratio = 300:900 = 1:3
losses caused directly by the indemnifier. Option a is
incorrect because an implied contract of indemnity
3. Correct Option A
does not automatically cover losses caused by third
(H & S)
parties. Option b is incorrect as prior knowledge of
Mobile phone sold on Monday
potential losses by a third party is not a requirement
 60 
for an implied contract of indemnity. Option d is    2700  450
 360 
incorrect as including third-party losses would need
Similarly, we can find the values of all the days.
to be explicitly agreed upon in the contract.
Days Phones
sold
Section V
Monday 450
PASSAGE I
Tuesday 300
1. Correct Option D
Wednesday 750
(H & S)
Thursday 300
Mobile phone sold on Monday
Friday 900
 60 
   2700  450
 360 
Sum of phones sold on Wednesday and Thursday
Similarly, we can find the values of all the days.
= 750 + 350 = 1050
Days Phones sold
Monday 450
Tuesday 300

[54]
4. Correct Option C PASSAGE II
(H & S) 6. Correct Option C
Mobile phone sold on Monday (H & S)
 60  Shop Rs.500 Rs. 200 Rs. 100
   2700  450
 360  notes notes Notes
Similarly, we can find the values of all the days. A 12 42 82
Days Phones B 8 38 76
sold C 22 52 48
Monday 450 D 32 24 44
Tuesday 300 E 14 18 66
Wednesday 750
Thursday 300 Total amount of Rs 500 notes with A and E
Friday 900 = (12+14 )500 = 26(500) = Rs 13000
Average mobile phones sold on Tuesday, Total amount of Rs 200 notes with B
Wednesday and Thursday = = 38(200) = Rs 7600
 300  750  300  7600
100% = 58.46 %
 450 Required % =
3 13000

5. Correct Option C 7. Correct Option A


(H & S) (H & S)
Mobile phone sold on Monday Shop Rs.500 Rs. 200 Rs. 100

 60  notes notes Notes


   2700  450
 360  A 12 42 82
Similarly, we can find the values of all the days. B 8 38 76
Days Phones C 22 52 48
sold D 32 24 44
Monday 450 E 14 18 66
Tuesday 300
Wednesday 750 Total amount of money C has
Thursday 300 = 22(500) + 52(200) + 48(100)
Friday 900 =11000 + 10400 + 4800 = Rs 26200
Amount of both the smartphones = 9200 + 4000

 450  = Rs 13200
Required % =   100% = 60%
 750  Money Left = 26200=13200 = Rs 13000
So, 15% of amount left with C = 0.15(13000)
= Rs 1950

[55]
8. Correct Option C Ratio of Rs 200 notes with D : F = 4:5
(H & S)  24 : F = 4:5
Shop Rs.500 Rs. 200 Rs. 100  Rs 200 notes with F = 30
notes notes Notes Therefore, total notes with F = 90+30 = 120
A 12 42 82 Average = 120/3 = 40
B 8 38 76
C 22 52 48 10. Correct Option D
D 32 24 44 (H & S)
E 14 18 66 Shop Rs.500 Rs. 200 Rs. 100
notes notes Notes
Ratio of the number of 200 and 100 rupee notes with A 12 42 82
shopkeeper E to the number of 500 rupee notes with B 8 38 76
shopkeepers B and C together C 22 52 48
= (18 + 66):(22 + 8) = 84:30 = 14:5 D 32 24 44
E 14 18 66
9. Correct Option B
(H & S) Total amount of Rs 500 with D
Shop Rs.500 Rs. 200 Rs. 100 = 32(500) = Rs 16000
notes notes Notes Total amount of Rs 100 with B = 76(100) = Rs 7600
A 12 42 82 Difference = 16000- 7600 = Rs 8400
B 8 38 76
C 22 52 48
D 32 24 44
E 14 18 66

Total Rs 500 and Rs 100 notes with C


=48 + 22 = 70
So,
Total Rs 500 and Rs 100 notes with

9
F=    70  90
7

For more questions, kindly visit the library section: Link for app: https://links.physicswallah.live/vyJw

For more questions, kindly visit the library section: Link for web: https://physicswallah.live/tabs/tabs/library-
tab
PW Mobile APP: https://physicswala.page.link/?type=contact-us&data=open
For PW Website: https://www.physicswallah.live/contact-us

[56]

You might also like